CEN Practice Test 6 with Rationale

CEN Practice Test 6 with Rationale

Taking timed CEN Exam Questions can help improve your test-taking speed.

BCEN Practice Test 6 with Rationale

Question 1.    
A 50-year-old man with chronic hypertension is brought to the emergency department with retrosternal chest pain. The patient was diagnosed hypertensive ten years ago but has not been compliant with medications. He is diagnosed with myocardial infarction, a form of acute coronary syndrome (ACS). What is the primary cause of ACS?
(a) Atherosclerosis.
(b) Vectors.
(c) Hematogenous spread.
(d) Genetic inheritance.
Answer:
(a) Atherosclerosis.

Atherosclerosis is the narrowing and hardening of arteries caused by the deposition of plaques in the arterial walls. It usually starts after birth and progressively worsens with age. Risk factors include an unhealthy diet, elevated cholesterol levels, smoking, diabetes, alcohol, genetics, and a sedentary lifestyle. Atherosclerosis is implicated in myocardial infarction because it contributes to plaque formation, eventually leading to the occlusion of the coronary artery, impaired blood flow to the heart, and myocardial ischemia.

Question 2.
A 58-year-old man comes to the ER with chest pain. He has no history of myocardial infarction, and ECG changes are nonspecific. Which factor would most suggest a diagnosis of unstable angina for this patient?
(a) Elevated cardiac troponin levels.
(b) A positive!result in an exercise stress test.
(c) The absence of elevated cardiac enzymes.
(d) ST-segment elevation on ECG.
Answer:
(c) The absence of elevated cardiac enzymes.

Unstable angina is a clinical syndrome that falls between stable angina and acute myocardial infarction. One of the differentiating factors between unstable angina and myocardial infarction is the absence of elevated cardiac enzymes like troponins in unstable angina. Elevated troponins would suggest myocardial injury, as seen in myocardial infarction. 

Question 3.
A 45-year-old man with a 10-year history of poorly controlled diabetes is rushed to the ER with complaints of retrosternal chest pain persisting for 30 minutes. There was associated diaphoresis, dyspnea, and pre-syncope. Which investigative tool can distinguish myocardial infarction from unstable angina in this patient?
(a) Cardiac enzymes.
(b) Chest X-ray.
(c) Abdominal USS.
(d) Serum Lipase.
Answer:
(a) Cardiac enzymes.

Cardiac enzymes help differentiate the different causes of acute coronary syndromes. Cardiac enzymes include troponins and creatinine kinase MB (CK-MB). Acute coronary syndrome is classified into unstable angina, ST-elevation myocardial infarction, and non- ST elevation myocardial infarction. The risk factors for acute coronary syndrome include diabetes, hypertension, and dyslipidemia. In this patient, the risk factor was poorly controlled diabetes. A chest X-ray, abdominal ultrasound, or serum lipase would not differentiate myocardial infarction from unstable angina. The diagnosis is made by testing for cardiac enzymes that determine unstable angina from myocardial infarction (cardiac enzymes are at high levels in myocardial infarction).

Question 4.
A middle-aged man is suspected to have suffered a myocardial infarction. Which of the following investigations helps distinguish ST-elevation myocardial infarction (STEMI) from non-ST-elevation myocardial infarction?
(a) Electrocardiogram.
(b) Chest X-ray.
(c) Abdominal ultrasound scan.
(d) Cardiac enzymes. 
Answer:
(a) Electrocardiogram.

An electrocardiogram helps to differentiate between ST-elevation myocardial infarction (STEMI) and non-ST-elevation myocardial infarction (NSTEMI). In a STEMI, there is persistent ST-segment elevation on the ECG due to the complete occlusion of a coronary artery. This means there is a more severe blockage. In an NSTEMI, the ECG may show ST-segment depression, T-wave inversion, or no significant changes, which means there is a partial blockage of the heart muscle. 

Question 5.    
A 52-year-old female presents with sudden severe chest pain radiating to her back. Upon examination, you notice a variation in blood pressure between her arms. This finding is most consistent with which of the following conditions?
(a) Myocardial infarction.
(b) Aortic dissection.
(c) Pulmonary embolism.
(d) Pericarditis.
Answer:
(b) Aortic dissection.

Aortic dissection can cause a difference in blood pressure between the two upper limbs due to the separation of layers in the aortic wall, which can compromise blood flow to one of the subclavian arteries. This difference in blood pressure is a classic clinical sign that can aid in diagnosing aortic dissection. Such blood pressure deficits between the upper limbs are not characteristic of the other listed conditions.

Question 6.
A hypertensive 68-year-old man with ulcerative colitis and a 78-year-old man with atherosclerosis were both diagnosed with an aortic aneurysm. Which of the following is not an associated risk factor or etiology of aortic aneurysm in these patients?
(a) Atherosclerosis.
(b) Severe systemic hypertension.
(c) Ulcerative colitis.
(d) Ehlers-Danlos syndrome.
Answer:
(c) Ulcerative colitis.

Ulcerative colitis is not a risk factor for aortic dissection or aneurysm. Aortic dissection and aneurysm are two clinical conditions that are related to pathophysiology. Aortic dissection refers to the accumulation of blood within the layers of the aorta, not the heart. Aortic aneurysm refers to dilating a portion of the aorta due to weakness in its walls. Both have similar risk factors, which include hypertension, atherosclerosis, smoking, and connective tissue disorders like Marfan syndrome and Ehlers-Danlos syndrome. Aortic dissection causes retrosternal chest pain radiating to the back, with blood pressure differences of more than 20 mmHg in the two upper limbs. A feared complication of aortic aneurysm is rupture and massive hemorrhage. Diagnosis is by CT aortography, which shows the dissection, aneurism, and widened mediastinum.

Question 7.    
A 70-year-old man developed a major complication from an abdominal aortic aneurysm. Which of the following is the most dreaded complication that could occur?
(a) Hypertension.
(b) Rupture.
(c) Embolization.
(d) Disseminated intravascular coagulopathy (DIC).
Answer:
(b) Rupture.

A feared complication of aneurysm is rupture and subsequent massive hemorrhage. A rupture can result in a stroke, coma, and even death. Other complications of abdominal aortic aneurysms include:

  • Fistula formation.
  • Occlusion of blood vessels from thrombus formation.
  • Compression of surrounding structures by the aneurysm which can lead to further disease manifestations.

Diagnosis is by CT aortography, which shows the dissection, aneurism, and a widened mediastinum. Aortic dissection generally refers to the accumulation of blood within the layers of the aorta. Risk factors include hypertension, atherosclerosis, smoking, and connective tissue disorders like Marfan syndrome and Ehlers-Danlos syndrome. Aortic aneurysms have also been shown to increase with age which is a risk factor in this patient.

Question 8.
What should be the first step an emergency nurse takes when a patient suddenly - collapses in front of the hospital? The patient was on her way to the outpatient clinic before she fell and is now found to be pulseless.
(a) Urgent CT scan.
(b) Cardiopulmonary resuscitation.
(c) Urgent chest X-ray.
(d) Urgent Echocardiography.
Answer:
(b) Cardiopulmonary resuscitation.

CPR can be started at the point of collapse and continued as the patient is moved to the ER for further treatment. It should be determined whether the cause of the sudden collapse is arrhythmia and whether it is a shockable rhythm. Shockable rhythms include ventricular fibrillation and ventricular tachycardia while non-shockable rhythms include pulseless electrical activity and asystole. 

Question 9.
Which clinical sign indicates an emergency nurse should begin cardiopulmonary resuscitation on a patient?
(a) Seizure episode.
(b) Pre-syncope.
(c) Pulselessness.
(d) Hypotension.
Answer:
(c) Pulselessness.

The indication for CPR in a patient includes sudden collapse and pulselessness on examination. At this point, the cause of the sudden collapse should be determined. If it is an arrhythmia, decide whether or not it is a shockable or non-shockable rhythm. Defibrillators are usually programmed to detect if it is a shockable or non-shockable rhythm. Shockable rhythms include ventricular fibrillation and pulseless ventricular tachycardia, while non-shockable rhythms are pulseless electrical activity and asystole.

Question 10.
In which of the following conditions is defibrillation contraindicated for a patient experiencing cardiac arrest?
(a) Pulseless ventricular tachycardia.
(b) Ventricular fibrillation.
(c) Asystole.
(d) All of the above.
Answer:
(c) Asystole.

Asystole is a condition in which defibrillation is contraindicated. The heart no longer pumps in asystole, and the contraction mechanism has shut down. It might result from a heart attack that was not attended to. Hence, the application of shock is not an effective solution. The shock might make it difficult for the patient to recover. The best thing to do in asystole is to commence CPR since this increases the patient’s chances of survival. Defibrillation is the application of electrical shock to restore sinus rhythm in patients with severe arrhythmias. Automated External Defibrillators (AEDs) are usually programmed to detect if it is a shockable or non-shockable rhythm. Shockable rhythms include ventricular fibrillation and pulseless ventricular tachycardia. Non-shockable rhythms are pulseless electrical activity and asystole.

Question 11.
A 50-year-old patient with chronic hypertension is brought to the emergency unit with several episodes of syncope, chest palpitations, fluttering, and fatigue. Their blood pressure reading is 110/80 mmHg, and their pulse is irregular. What should be the emergency nurse’s initial line of treatment for this patient?
(a) Rate and rhythm control.
(b) Defibrillation.
(c) Heart failure regimens.
(d) Diuretics.
Answer:
(a) Rate and rhythm control.

Treatment of atrial fibrillation can either control the heart rate (using beta blockers, calcium channel blockers) or the rhythm (using amiodarone). Atrial fibrillation presents with chest palpitations and pre-syncope, possibly leading to syncopal episodes. Atrial fibrillation occurs from a focus in the ostium of the pulmonary veins.

Question 12.
Which statement correctly differentiates the ECG of a 50-year-old patient with atrial fibrillation from the ECG of a 60-year-old patient with atrial flutter?
(a) Atrial flutter is associated with absent P waves.
(b) Atrial flutter shows a sawtooth pattern on EKG.
(c) Atrial fibrillation is associated with regular R-R intervals.
(d) Atrial flutter is associated with irregular R-R intervals.
Answer:
(b) Atrial flutter shows a sawtooth pattern on EKG.

Atrial fibrillation and flutter are two similar forms of supraventricular tachycardia and must be differentiated. Atrial fibrillation occurs from a focus in the ostium of the pulmonary veins. Atrial flutter originates in the right atrium. Atrial fibrillation causes chest palpitations and syncope in severe cases, similar to atrial flutter. However, an electrocardiogram helps to differentiate them. The absence of P waves and irregular R-R internal characterizes atrial fibrillation. Atrial flutter causes a sawtooth appearance on the electrocardiogram. Both are associated with tachycardia on an EKG. 

Question 13.
Which statement accurately characterizes a first-degree heart block in a 45-year-old man?
(a) Prolonged PR interval of greater than 0.2 seconds.
(b) Progressive PR interval prolongation with skipped beats.
(c) Regular PR interval prolongation leading to a skipped beat.
(d) Complete dissociation between the P waves and the QRS complexes.
Answer:
(a) Prolonged PR interval of greater than 0.2 seconds.

First-degree heart block is due to a blockage in the AV-node conduction. It is characterized by PR interval prolongation of more than 0.2 seconds. This PR interval should typically be from 0.12 to 0.20 seconds. For example, when it goes above 0.3 seconds, the heart experiences what is a marked first-degree block. Fundamentally, the PR interval refers to the time it takes the electrical impulse generated from the Sinoatrial (SA) node to propagate from the atria to the Purkinje fibers. The block can be caused by Myocardial Infarction, electrolyte abnormalities, Atrioventricular node anomalies, or even medications. Other types of heart block include second-degree block (Mobitz I and Mobitz type II) and third-degree heart block, characterized by total dissociation between the atrial beats and ventricular beats.

Question 14.
A 55-year-old man who presented with breathlessness and easy fatigability was diagnosed with a third-degree heart block. Which of the following statements is a correct description of his condition?
(a) PR interval prolongation of more than 0.2 seconds, with no missed beats.
(b) PR interval progressively extends with each round, leading to a missed beat.
(c) PR interval prolongation, leading to a skipped beat.
(d) Complete dissociation between the P waves and the QRS complexes. 
Answer:
(d) Complete dissociation between the P waves and the QRS complexes.

Heart blocks are arrhythmias which are caused by blockade in the electrical conduction system of the heart at different levels. There are different types, including first-degree, second-degree,<and third-degree heart block. Third-degree or complete heart block is characterized by total dissociation between the atrial and ventricular beats. Here, the abnormal heart rhythm that is seen results from an anomaly in the heart’s conducting system. On ECG, the P waves and QRS complexes are in complete desynchrony. The QRS rates are being propagated independently of the P waves. There is also a preponderance of P waves than QRS complexes. However, P-P and R-R intervals might also be regular in a third-degree heart block. This condition can either be from birth or acquired. When acquired, there is a wide range of causes, including, but not limited to, medications, degenerative diseases, rheumatic diseases, infections, and metabolic causes.

Question 15.
Which of the following statements correctly differentiates Mobitz type I second-degree heart block from Mobitz type II?
(a) Mobitz type I is associated with progressive prolongation of the PR interval.
(b) Mobitz type II is associated with progressive prolongation of the PR interval.
(c) Mobitz type I is associated with progressive prolongation of the QRS complex.
(d) Mobitz type II is associated with progressive prolongation of the QRS complex.
Answer:
(a) Mobitz type I is associated with progressive prolongation of the PR interval.

Mobitz type I is characterized by progressive P-R interval prolongation and eventually dropped beat, whereas Mobitz type II is characterized by fixed P-R interval and dropped beat. Heart blocks are a form of arrhythmias which are caused by blockage in the heart’s electrical conduction system at different levels. There are different types, including first-degree, second-degree, and third-degree heart block. A second-degree block is divided into two: Mobitz I and Mobitz type II. Most of the cases of Mobitz I are due to anomalies in the Atrioventricular node, while most of the cases of Mobitz II are due to conduction disturbances infranodally. Mobitz II also poses a higher risk to patients than Mobitz I, although it is less common than Mobitz I. 

Question 16.
An emergency nurse notices a surge in patients with subacute infective endocarditis. Which of the following counseling actions by the emergency nurse can reduce the burden of infective endocarditis in the population?
(a) Education on adequate blood pressure control.
(b) Education on oral hygiene.
(c) Education on routine chest X-ray.
(d) Education on routine chest CT scan.
Answer:
(b) Education on oral hygiene.

Infective endocarditis refers to inflammation of the inner lining of the heart. It is divided into acute and subacute infective endocarditis. Acute infective endocarditis can be caused by Staphylococcus aureus, especially in patients with normal heart valves, but other organisms can also cause it. Risk factors include intravenous drug use. Subacute endocarditis is common in people with poor oral hygiene and is caused by Viridans streptococci. Subacute endocarditis occurs in people with damaged or prosthetic heart valves. A clinical sign to diagnose endocarditis is the presence of a new murmur on auscultation. Treatment is by antibiotics.

Question 17.
Staphylococcus aureus causes which type of infective endocarditis?
(a) Acute infective endocarditis.
(b) Subacute endocarditis.
(c) Libman-Sacks endocarditis.
(d) Duke’s endocarditis.
Answer:
(a) Acute infective endocarditis.

Staphylococcus aureus is a common cause of acute infective endocarditis, but it’s not the only organism responsible. Acute infective endocarditis is inflammation of the endocardium in a patient with competent heart valves. The most common risk factor for acute infective endocarditis is intravenous drug abuse. Intravenous drug abuse causes dissemination of skin flora into the bloodstream and subsequent infection of the endocardium. Treatments include investigations to confirm diagnosis and intravenous antibiotics.

Question 18.
What is a major criterion in the Duke’s criteria for diagnosing acute infective endocarditis?
(a) High-grade fever on examination.
(b) Immunologic findings such as Osier’s nodes, Roth spots, and glomerulonephritis.
(c) Vascular phenomena such as Janeway lesions.
(d) Two positive blood cultures of implicated organisms.
Answer:
(d) Two positive blood cultures of implicated organisms.

Definitive diagnosis of infective endocarditis involves two positive blood cultures for implicated organisms. In addition to blood culture, echocardiography is also important in confirming the diagnosis of infective endocarditis. Echocardiography confirms the presence of vegetation and valvular regurgitation. The two, coupled together, constitute the major Duke’s criteria. The other criteria include splinter hemorrhage, Osier nodes, and Janeway lesions.

Question 19.
An emergency nurse is evaluating a 36-year-old patient for infective endocarditis. Using Duke’s criteria for diagnosis of infective endocarditis, it is possible to pick one of the two major criteria on clinical examination. Which of the following aspects of cardiovascular examination is the most suggestive clinical sign of infective endocarditis?
(a) Inspection.
(b) Palpation.
(c) Percussion.
(d) Auscultation. 
Answer:
(d) Auscultation.

Cardiovascular auscultation can diagnose infectious endocarditis, which demonstrates valvular regurgitation features. The valvular regurgitation murmurs suggest infective endocarditis in a patient with fever and heavy sweating at night.

Question 20.
A 50-year-old hypertensive man who had mitral valve replacement five years ago is brought into the emergency unit with complaints of progressive fatigue, low-grade fever, and night sweats for the past few days. Chest auscultation reveals a new onset murmur. Echocardiogram and blood cultures confirm the diagnosis of infective endocarditis. Which of the following is a major risk factor for this patient’s condition?
(a) Prosthetic heart valves.
(b) Age.
(c) Hypertension.
(d) Sex.
Answer:
(a) Prosthetic heart valves.

Subacute endocarditis is common in people with poor oral hygiene and is caused by Viridans streptococcus. Subacute endocarditis occurs in people with damaged or prosthetic heart valves. 

Question 21.
A nursing student in the emergency department is evaluating a patient with congestive heart failure. Upon auscultation, an additional heart sound is detected. The senior nurse confirms this finding is typical in heart failure patients. What is the most likely abnormal clinical finding?
(a) S4.
(b) S3.
(c) S1.
(d) S2.
Answer:
(b) S3.

Heart failure is the inability of the heart to pump blood to meet the metabolic demands, or it is doing so by increasing the filling pressure. Decompensated heart failure presents with dyspnea, orthopnea, and pedal swelling. The examination shows distended neck veins and an S3 added heart sound. Angiotensin Converting Enzyme Inhibitors (ACEIs) like Lisinopril have been shown to improve survival rates in patients with heart failure. Other drugs used are Furosemide and Digoxin. Decompensated left ventricular failure may present with acute cardiogenic pulmonary edema and is treated with Furosemide.

Question 22.
A 28-year-old chemical lab technician arrives in the ER with intense eye pain and blurred vision after accidentally splashing a chemical in her eye. Which of the following is the most appropriate initial step for the nurse?
(a) Administer antibiotic eye drops.
(b) Test visual acuity.
(c) Begin immediate and copious eye irrigation.
(d) Apply a warm compress over the eye.
Answer:
(c) Begin immediate and copious eye irrigation.

In cases of chemical eye contamination, the priority is to reduce the contact time between the chemical and the ocular surface. Immediate and copious irrigation helps to dilute and remove the chemical, thereby reducing the potential for injury. Although other steps, such as checking visual acuity or administering medications might be necessary later, the primary goal initially is to minimize damage by irrigating the eye.

Question 23.
Which drug is most useful in managing acute pulmonary edema following left ventricular failure?
(a) Lisinopril.
(b) Furosemide.
(c) Digoxin.
(d) Verapamil. 
Answer:
(b) Furosemide.

Furosemide is a loop diuretic that acts by inhibiting the Na/k/2cl transporter in the thick ascending loop of Henle. It causes inhibition of salt and water reabsorption, leading to hypotonic urine. Loss of excess water in urine leads to a reduction in cardiac preload. Reducing cardiac preload reduces pulmonary and systemic congestive symptoms seen in heart failure.

Question 24.
A 55-year-old patient with decompensated heart failure is brought to the emergency department. The emergency nurse assesses the patient. Which of the following drugs have been shown to increase survival rate in patients with chronic heart failure?
(a) Lisinopril.
(b) Furosemide.
(c) Digoxin.
(d) Verapamil.
Answer:
(a) Lisinopril.

Lisinopril is an angiotensin-converting enzyme inhibitor (ACEI) that works by inhibiting the conversion of angiotensin I to angiotensin II. It leads to a reduction in angiotensin II levels. Angiotensin II helps raise the blood pressure and increases salt and water reabsorption. ACEIs, therefore, decrease blood pressure and salt and water reabsorption, reducing preload. Studies have shown that ACEIs and angiotensin receptor blockers (ARBs) increase survival in heart failure patients.

Question 25.
Which of the following conditions causes heart failure with reduced ejection fraction?
(a) Dilated cardiomyopathy.
(b) Hypertrophic cardiomyopathy.
(c) Restrictive cardiomyopathy.
(d) Cardiac arrest.
Answer:
(a) Dilated cardiomyopathy.

Dilated cardiomyopathy is a heart disease caused by mutations in cardiac proteins like myosin and actin or acquired conditions like chronic alcoholism and drugs. It leads to dilation of the ventricular chambers and reduced cardiac output. Progressive maladaptation may lead to heart failure with reduced ejection fraction. 

Question 26.
A 45-year-old man is brought to the emergency nurse complaining of chest pain radiating to his back. The patient has no history of medical disorders and takes no medications. Echocardiography shows fluid around the heart. What is a possible complication of pericardial effusion that the nurse should be wary of?
(a) Heart failure.
(b) Cardiac arrest.
(c) Cardiac tamponade.
(d) Cardiac arrhythmia.
Answer:
(c) Cardiac tamponade.

Pericardial effusion is the accumulation of fluid in the pericardium of the heart. Accumulation of large volumes of fluid can compress the heart, leading to a condition called cardiac tamponade. Cardiac tamponade is a severe condition that causes a restriction to the ability of the heart to pump blood to meet the body’s metabolic demands. The result is a reduction in cardiac output and shock in severe cases. Common signs and symptoms of cardiac tamponade include dyspnea, chest pain, tachycardia, jugular venous distension, and hypotension. Cardiac tamponade requires early diagnosis and treatment by pericardiocentesis.

Question 27.
A 55-year-old man with chronic hypertension develops acute viral pericarditis. Which of the following drugs is approved for use in treating acute pericarditis with minimal effusion?
(a) Thiazides.
(b) Beta-blockers.
(c) Diuretics.
(d) NSAIDs.
Answer:
(d) NSAIDs.

Acute pericarditis is the inflammation of a layer of the heart known as the pericardium. Acute pericarditis causes chest pain that radiates to the back and is relieved by leaning forward. Acute pericarditis can arise from infections, myocardial ischemia, and inflammatory disorders. Treatment depends on etiology and severity. In most cases, the treatment involves NSAIDs for the pain and the treatment of the underlying etiology.

Question 28.
A 55-year-old patient with chronic kidney disease, a myocardial infarction six months prior, and a viral infection one week prior is diagnosed with acute pericarditis. He takes Lisinopril in addition to other drugs for his condition. Which of the following is not a known etiology/risk factor for developing acute pericarditis and pericardial effusion?
(a) Myocardial infarction.
(b) Coxsackie virus infection.
(c) Lisinopril.
(d) Uremia. 
Answer:
(c) Lisinopril.

Lisinopril is an angiotensin-converting enzyme (ACE) inhibitor commonly used to treat hypertension and heart failure. Acute pericarditis is the inflammation of a layer of the heart known as the pericardium. Acute pericarditis causes chest pain that radiates to the back and is relieved by leaning forward. Acute pericarditis can arise from infections, myocardial ischemia, and inflammatory disorders. Treatment depends on etiology and severity. In most cases, the treatment involves NSAIDs for the pain and the treatment of the underlying etiology.

Question 29.
A 35-year-old woman with no previous medical history is brought to the emergency department with a 2-hour onset of central chest pain that radiates to her back. There is associated generalized weakness and a low-grade fever. The patient said she recently developed an upper respiratory tract infection, which was treated with over-the-counter medications. Blood pressure is 120/60 mmHg, and temperature is ioo°F (37.8°C). Cardiac enzymes are within normal limits. What is the most likely diagnosis?
(a) Uremic pericarditis.
(b) Viral pericarditis.
(c) Dressler’s syndrome.
(d) Heart failure.
Answer:
(b) Viral pericarditis.

The patient’s presentation with acute onset chest pain, low-grade fever, and recent upper respiratory tract infection suggests viral pericarditis. Acute pericarditis is the inflammation of a layer of the heart known as the pericardium. Acute pericarditis causes chest pain that radiates to the back and is relieved by leaning forward. Acute pericarditis can arise from infections, myocardial ischemia, and inflammatory disorders. Treatment depends on etiology and severity. In most cases, the treatment involves NSAIDs for the pain and the treatment of the underlying etiology. 

Question 30.
What is the best next step for managing a patient being treated for myocardial infarction who has now developed cardiac tamponade?
(a) NSAIDs.
(b) Colchicine.
(c) Chest tube.
(d) Corticosteroids.
Answer:
(c) Chest tube.

Cardiac tamponade is a medical emergency where fluid accumulates in the pericardial space, which leads to increased pressure on the heart and impairs its ability to pump blood. The most effective immediate treatment for cardiac tamponade is removing the fluid. This is typically done by pericardiocentesis or the insertion of a chest tube. NSAIDs, colchicine, and corticosteroids can be used to manage pericarditis, but they do not address the immediate life-threatening situation of cardiac tamponade. Immediate relief from the increased pericardial pressure is achieved by draining the fluid.

Question 31.
A 15-year-old boy develops sudden pain, pallor, and cyanosis of the digits during winter. He is diagnosed with Raynaud’s disease. Which of the following statements is correct about Raynaud’s phenomenon?
(a) It presents with digital pain, cyanosis, and paresthesia.
(b) It presents with digital edema.
(c) It presents with digital plethora.
(d) It presents with bounding pulses in the hands and feet.
Answer:
(a) It presents with digital pain, cyanosis, and paresthesia.

Raynaud’s disease is a medical condition characterized by episodes of vasospasm of the small arteries and arterioles, typically in digits. It is often exacerbated by cold temperatures or emotional stress. The affected limbs are usually pale blue and may show features of ulceration. During these episodes, patients may also present with pain, numbness, and tingling (paresthesia). Raynaud’s phenomenon is often a benign and self-limited condition. In some cases, it may be associated with underlying connective tissue disorders such as systemic lupus erythematosus or scleroderma.

Question 32.
Which of the following is not correct about peripheral arterial disease?
(a) It is caused by atherosclerosis and the narrowing of the vessels.
(b) It presents with intermittent claudication in the limbs that is worsened by activity.
(c) It presents with numbness and tingling sensations in the limbs.
(d) It is caused by the degeneration of the nerve supply to the limbs.
Answer:
(d) It is caused by the degeneration of the nerve supply to the limbs.

Peripheral arterial disease is caused by occlusion of the blood vessels of the limbs due to atherosclerosis. The narrowing of the vessels leads to a reduction in blood flow and ischemia, especially during exercise. Patients experience pain, paresthesia, cold and pale limbs, and intermittent claudication.

Question 33.
Which of the following can trigger the symptoms of Raynaud’s disease?
(a) Fever.
(b) Cold.
(c) Diabetes.
(d) Chronic kidney disease.
Answer:
(b) Cold.

Cold is a common trigger for Raynaud’s disease. This condition is characterized by vasospasm and reduced blood flow in the limbs in response to triggers. Exposure to cold temperatures is a common trigger. Due to reduced blood flow, the affected digits often turn pale (pallor) and blue (cyanosis). As the vasospasm resolves and blood flow returns, the digits may turn red (erythema). Other triggers for Raynaud’s disease may include emotional stress, vibrating tools or equipment, and smoking. 

Question 34.
A 50-year-old man with a recent history of long-distance travel is brought to the emergency room due to progressive swelling and pain in the right lower limb. The patient has a 35-year smoking history and was previously admitted for deep venous thrombosis. Which of the following will help the nurse confirm a diagnosis of deep venous thrombosis in this patient?
(a) Doppler compression ultrasound.
(b) X-ray of the limb.
(c) Bleeding time.
(d) Clotting time.
Answer:
(a) Doppler compression ultrasound.

Doppler compression ultrasound is an investigation modality for confirming deep venous thrombosis (DVT) in patients with suspected lower extremity thrombosis. DVT is a medical condition caused by the formation of thrombosis, usually in the veins of the pelvis and limbs. If not treated, DVT can be life-threatening. It can lead to the formation of acute pulmonary embolism, a life-threatening condition characterized by acute chest pain, dyspnea, and cardiopulmonary compromise. Doppler ultrasound uses sound waves to detect blood flow and blood clots in the veins. In the case of DVT, the ultrasound can identify the presence of a blood clot in the affected deep vein and determine its size and location. Diagnostic tests used with Doppler ultrasound include a D-dimer blood test, venography, and magnetic resonance venography (MRV).

Question 35.
Which of the following helps prevent acute pulmonary embolism in patients with deep venous thrombosis?
(a) Inferior vena cava filter.
(b) Antibiotics.
(c) Analgesics.
(d) Antihypertensive.
Answer:
(a) Inferior vena cava filter.

An inferior vena cava filter helps prevent acute pulmonary embolism in patients with a history of deep vein thrombosis. An IVC filter is placed to prevent the passage of a clot through the inferior vena cava to the right chambers of the heart, where it can travel to the lung and cause acute pulmonary embolism. Anticoagulation is also used.

Question 36.
Which of the following describes a condition which presents with tortuous dilatation of the veins in the lower limb?
(a) Deep venous thrombosis.
(b) Varicose veins.
(c) Arterial thrombosis.
(d) Vein occlusion.
Answer:
(b) Varicose veins.

Varicose veins are tortuous dilatations of the veins of the lower limb. The veins are prominent and usually found on the legs, not the hands or feet. It occurs due to incompetent venous valves, which lead to blood pooling and tortuous dilation. The varicose veins can become injured, and the ulcers can get infected. Another complication is thrombophlebitis.

Question 37.
A 40-year-old driver is brought to the emergency room after he hit his chest against the wheel and suffered a chest trauma injury. Which of the following is a complication of cardiac contusion due to blunt trauma?
(a) Arrhythmia.
(b) Hypertension.
(c) DIC.
(d) All of the above.
Answer:
(a) Arrhythmia.

Cardiac arrhythmia is a common complication of cardiac contusion secondary to blunt chest trauma. Cardiac contusion can occur after a road traffic accident or a fall from a height. The force of injury can disrupt the heart’s electrical activity, which can lead to arrhythmias. Arrhythmias are abnormal heart rhythms. They are classified as supraventricular tachycardia (fast heart rate), bradycardia (slow heart rate), ventricular abnormalities, or other abnormal rhythms. Patients with cardiac contusion require resuscitation, pain management, and electrocardiogram monitoring to prevent sudden cardiac death. 

Question 38.
Which of the following correctly describes obstructive shock?
(a) A reduction in the vascular volume.
(b) A severe inflammatory response to infection.
(c) Hypertensive heart disease.
(d) The heart fails to fill and empty because of extrinsic factors.
Answer:
(d) The heart fails to fill and empty because of extrinsic factors.

Shock results when there is a failure of the heart to pump blood to meet the metabolic demands of the body. Shock is classified into several types, which include cardiogenic, obstructive, septic, and anaphylactic. Obstructive shock occurs when there is an obstruction to cardiac emptying due to extrinsic compression. The cause of the extrinsic compression may be a mass, cardiac tamponade, or acute pulmonary embolism. Like all other causes of shock, the treatment for extrinsic shock is acute resuscitation and correction of underlying etiology.

Question 39.
Which of the following is the next best step in managing a 40-year-old man who developed tension pneumothorax complicated by obstructive shock in the emergency room?
(a) Needle thoracentesis.
(b) Intravenous fluids.
(c) Diuretics.
(d) Analgesics.
Answer:
(a) Needle thoracentesis

Tension pneumothorax is a medical emergency characterized by a sufficient quantity of gas accumulation in the pleural space, which then causes cardiopulmonary compromise. The immediate management for a tension pneumothorax is needle thoracentesis (also known as needle decompression) to release the trapped air and relieve the pressure. This is followed by a chest tube placement. Tension pneumothorax can frequently complicate penetrating chest traumas and should be evaluated for and treated in patients with trauma. The ATLS protocol outlines the management of trauma. The protocol lists common causes of death following trauma and trauma treatments. The common causes of death in the immediate post-traumatic period include massive hemothorax, tension pneumothorax, cardiac tamponade, open pneumothorax, and flail chest.

Question 40.
A 65-year-old diabetic woman with poor glycemic control is brought to the emergency room after suffering a myocardial infarction complicated by shock. Which of the following is shock caused by acute myocardial infarction?
(a) Cardiogenic shock.
(b) Obstructive shock.
(c) Septic shock.
(d) Anaphylactic shock.
Answer:
(a) Cardiogenic shock.

Cardiogenic shock following myocardial infarction is a phenomenon that occurs when a necrosed myocardium loses its contractile strength and fails to supply blood to meet the metabolic demands of the body. This leads to an accumulation of blood in the left ventricular. The result is pulmonary edema and shock. Features of cardiogenic shock include congestion, distended neck veins, and reduced cardiac output. There is also elevated pulmonary capillary wedge pressure. Management of cardiogenic shock includes inotropes and the management of pulmonary congestion.

Question 41.
A 75-year-old woman is brought to the emergency room after collapsing in her home. After acute resuscitation, the nurse is concerned about the risk of developing aspiration syndrome and takes necessary preventive actions. Which of the following conditions increases the risk of developing aspiration pneumonia?
(a) Unconsciousness.
(b) Severe hypertension.
(c) Bleeding disorders.
(d) Asthma.
Answer:
(a) Unconsciousness.

Unconsciousness is a risk factor for aspiration. Aspiration is also common in patients with neurologic impairments, such as motor neuron disease and multiple sclerosis. The aspirated material usually deposits in the inferior or posterior segments of the lungs. It is common in the right lung because of a more vertical right main bronchus. Diagnosis can be made with a chest X-ray and chest CT. It is treated with antibiotics and invasive procedures when implicated. 

Question 42.
Which of the following drugs is the most effective treatment for anaerobic infections above the diaphragm?
(a) Metronidazole.
(b) Clindamycin.
(c) Doxycycline.
(d) Amoxicillin.
Answer:
(b) Clindamycin.

Aspiration of oral secretions and food into the airway is common in patients with risk factors such as unconsciousness, delirium, neurologic diseases, and elderly patients. Aspiration increases the likelihood of aspiration pneumonia and lung abscesses. Aspiration pneumonia and lung abscesses are usually comprised of mixed organisms (aerobic and anaerobic infections). Clindamycin is used for anaerobic infection above the diaphragm.

Question 43.
A six-year-old girl is brought to the emergency room after an unknown object enters her airway. Aspiration of a foreign body is most likely to be dislodged in which part of the airway?
(a) Right lung.
(b) Left Lung.
(c) Pharynx.
(d) Larynx. 
Answer:
(a) Right lung.

Because of the peculiar anatomy of the respiratory tract, aspiration most commonly involves the right lung. This is because the right main bronchus is shorter and more vertical than the left, which is longer and more horizontal. Thus, a foreign body from the trachea is more likely to pass through the right main bronchus when it gets to the carina.

Question 44.
A 20-year-old known asthmatic woman is brought to the emergency room for a follow-up of her bronchial asthma. She is counseled on bronchial asthma, avoidance of exacerbation triggers, and the use of her inhaler. Which of the following is a known risk factor for developing bronchial asthma?
(a) Female gender.
(b) Personal history of autoimmune disorders.
(c) First-degree family history of asthma.
(d) Second-degree family history of asthma.
Answer:
(c) First-degree family history of asthma.

Bronchial asthma occurs more often in people with a first-degree family history of this diagnosis. Bronchial asthma is a type one hypersensitivity reaction of the airways, which presents with airway hyperresponsiveness, wheezing, cough, and dyspnea. Patients with mild to moderate cases experience chest tightness, breathlessness, wheezing, and cough. Symptoms are often worse during sleep. Oh presentation, signs include wheezing, tachypnea, pulsus paradoxus, tachycardia, and breathlessness, evidenced by the use of the accessory muscles of respiration.

Question 45.
A 25-year-old known asthmatic woman is brought to the emergency room after her breathing difficulty worsens and does not respond to her Ventolin inhaler. She speaks only in short phrases and is in severe respiratory distress. Which of the following is the diagnosis?
(a) Status epilepticus.
(b) Cardiac asthma.
(c) Status’asthmaticus.
(d) Chronic obstructive pulmonary disease.
Answer:
(c) Status asthmaticus.

Status asthmaticus refers to the worsening of severe acute exacerbation of asthma „ symptoms that are not relieved by the usual medications. Symptoms include wheezing, tachypnea, pulsus paradoxus, tachycardia, and breathlessness, evidenced by the use of the accessory muscles of respiration. Patients with severe exacerbations can present with altered consciousness, cyanosis, and a silent chest. In chronic cases, patients have barrel-shaped chests and hyperinflated lungs.

Question 46.
A six-year-old boy with bronchial asthma is newly diagnosed. The mother wants to know the complications of delayed treatment. Which of the following is a possible complication of acute severe asthma?
(a) Heart failure.
(b) Tension pneumothorax.
(c) Cardiac tamponade.
(d) Respiratory failure.
Answer:
(d) Respiratory failure.

Respiratory failure is a common complication seen in patients with status asthmaticus.
Due to persistent airway obstruction and mucus plugging, patients with acute severe asthma may develop respiratory failure. Patients with imminent signs of respiratory failure are transferred to the intensive care unit and managed with mechanical ventilation. 

Question 47.
An ER nurse is educating a patient about home-eye safety. Which of the following statements by the patient suggests a misunderstanding about the prevention of eye contamination diseases?
(a) “I should wear protective eyewear when working with chemicals.”
(b) “Regularly washing my hands can prevent eye infections.”
(c) “It’s okay to share my contact lens solution with family members.”
(d) “Expired eye drops should be discarded to avoid contamination.” 
Answer:
(c) “It’s okay to share my contact lens solution with family members.”

Sharing personal items like contact lens solution, makeup, or towels can spread eye infections or cause contamination. This statement indicates a misunderstanding and a potential risk for eye contamination disease.

Question 48.
During rounds, a nursing student asks the emergency room nurse about emphysema. Which of the following statements correctly describes emphysema?
(a) A chronic obstructive pulmonary disease characterized by chest pain and dizziness.
(b) A chronic obstructive pulmonary disease characterized by a persistent cough lasting three months for two consecutive years.
(c) A chronic obstructive pulmonary disease characterized by the destruction and degradation of the alveolar walls.
(d) A chronic obstructive pulmonary disease characterized by hypersensitivity to allergens.
Answer:
(c) A chronic obstructive pulmonary disease characterized by the destruction and degradation of the alveolar walls.

Emphysema is the progressive destruction of the lung parenchyma with loss of elastic recoil and destruction of alveolar walls and septa. Patients with emphysema are typically referred to as pink puffers, with a cachectic appearance, pursed-lip breathing, dome¬shaped chests, and use of the accessory muscles of respiration. In contrast, Chronic bronchitis is characterized by a chronic productive cough that lasts for at least three months in two consecutive years. Smoking is the predominant risk factor for chronic bronchitis. Patients with chronic bronchitis are referred to as blue bloaters with characteristic cyanosis, edema, chronic productive cough, leg swelling, and pulmonary hypertension.

Question 49.
A five-year-old girl is brought to the emergency room with complaints of dyspnea and irritability. A chest X-ray shows areas of lobar opacity. Which of the following statements correctly describes community-acquired pneumonia?
(a) Pneumonia contracted in a hospital setting 48-72 hours after admission.
(b) Pneumonia contracted outside of the hospital setting.
(c) Pneumonia contracted by prolonged endotracheal intubation.
(d) Pneumonia contracted immediately after admission.
Answer:
(b) Pneumonia contracted outside of the hospital setting.

Community-acquired pneumonia is contracted outside the hospital. The most common causative pathogens are streptococcus pneumonia, Haemophilus influenzae, atypical bacteria like Legionella species, chlamydia pneumonia, viruses, and Mycoplasma pneumoniae. Nosocomial pneumonia is hospital-acquired pneumonia. Risk factors include immunosuppression, intubation, old age, immobility, and sepsis. Nosocomial pneumonia is contracted by inpatients 48 to 72 hours after being admitted. It is generally caused by a bacterial infection rather than a virus. Implicated bacteria include rod-shaped, gram-negative organisms like Pseudomonas aeruginosa, Klebsiella pneumoniae, and Enterobacter spp; gram-positive bacteria like Staphylococcus aureus; and Haemophilus influenzae. Implicated viruses include influenza, respiratory syncytial virus, and cytomegalovirus.

Question 50.
A 30-year-old man is brought to the emergency room with complaints of a fever, a cough that produces foul-smelling sputum, and dyspnea. Chest CT shows features in keeping with empyema thoracis. Which of the following is not a risk factor for developing empyema thoracis?
(a) Pulmonary tuberculosis.
(b) Chronic bacterial pneumonia.
(c) Suppurative lung diseases.
(d) Cardiac asthma.
Answer:
(d) Cardiac asthma.

Empyema thoracis is not caused by cardiac asthma. Empyema thoracis is characterized by the collection of pus within the pleural cavity. Empyema thoracis is frequently a complication of pneumonia. It presents with an accumulation of abscesses in the pleural cavity. It occurs due to the spread of infection from the lung parenchyma to the pleural space. Causes include tuberculosis and pneumonia. Cardiac asthma refers to wheezing seen in patients with acute pulmonary edema. It does not cause empyema thoracis. 

Question 51.
Which of the following is an absolute indication for admission of a patient with a burn injury?
(a) Superficial burns.
(b) Involvement of the thighs.
(c) Inhalational injury.
(d) All of the above. 
Answer:
(c) Inhalational injury.

Inhalational injury is an absolute indication for admission following a burn injury. The presence of black soot in the nostrils indicates a possible inhalation injury, which can be serious and may lead to respiratory complications. Additionally, his lethargy and dyspnea raise concerns about potential airway compromise and systemic effects of smoke inhalation. The patient should be admitted for close monitoring, observation, and appropriate treatment to address any respiratory issues or systemic complications that may arise.

Question 52.
A 45-year-old man is brought to the emergency room after being involved in a fire. He is confused and suffered a partial thickness injury to the limbs. There is black soot in his nostrils, and he has laryngeal edema. Which of the following is the best next step?
(a) Oxygen by face mask.
(b) Oxygen by nasal cannula.
(c) Endotracheal intubation.
(d) Intravenous fluids.
Answer:
(c) Endotracheal intubation.

Loss of nasal hairs and soot in the nasal passages are clinical findings that should alert the nurse to the possibility of inhalation injury in a burn victim. The patient is evaluated via a focused history and examination. The patient is resuscitated via the ABC of resuscitation. When an inhalational injury is suspected, irrespective of the burn’s surface area or depth, the patient should be admitted and managed on an inpatient basis, where they can receive airway management.

Question 53.
Which of the following is the best initial therapy for a patient with an inhalational injury?
(a) Intravenous fluids.
(b) Oxygen therapy.
(c) Antibiotics.
(d) Cough suppressant.
Answer:
(b) Oxygen therapy.

Oxygen therapy is part of the initial approach for a patient with an inhalational injury. Mechanical ventilation may be required based on the severity of the injury and the patient’s respiratory status. Loss of nasal hairs and soot in the nasal passages are clinical findings that should alert the nurse to the possibility of an inhalation injury.

Question 54.
A 36-year-old man is brought to the emergency room following three weeks of insidious productive dry cough that has not responded to cough suppressants. The patient recently underwent radiotherapy for breast cancer and developed a cough two weeks after stopping radiotherapy. The patient was told he has a restrictive lung disease. Which of the following conditions cause a restrictive pattern of pulmonary disease?'
(a) Chronic asthma.
(b) Chronic bronchitis.
(c) Emphysema.
(d) Pulmonary fibrosis.    
Answer:
(d) Pulmonary Fibrosis.

Unlike chronic obstructive pulmonary disease, which causes an obstructive pattern, pulmonary fibrosis causes a restrictive pattern. Causes of pulmonary fibrosis include idiopathic pulmonary fibrosis, an inflammatory condition, and diffuse infiltrating disorders.

Question 55.
A 28-year-old patient with bronchial asthma was brought to the emergency room following an episode of acute asthma exacerbation while she was away from home and without her inhaler. Which of the following drugs is used to relieve the acute exacerbation of asthma?
(a) Formoterol.
(b) Salbutamol.
(c) Fluticasone.
(d) Salmeterol.
Answer:
(b) Salbutamol.

Salbutamol is a beta-2-receptor agonist (short-acting) used in the management of asthma exacerbations. It is a reliever of acute flares because of its short onset and duration of action. The other medications mentioned are longer-acting and are not used as relievers. 

Question 56.
Which of the following is a possible etiology for pleural effusion?
(a) Asthma.
(b) Chronic obstructive pulmonary disease.
(c) Idiopathic pulmonary fibrosis.
(d) Malignancy. 
Answer:
(d) Malignancy.

Pleural effusion is an accumulation of fluid within the pleural space. Causes are varied and classified as either transudates or exudates. Malignancy is one of the causes of exudative pleural effusion. Transudates may be caused by heart failure and nephrotic syndrome. The diagnosis is clinical and confirmed by a chest X-ray. However, thoracocentesis and analysis of the pleural fluid are needed to diagnose the cause. Treatment modalities include thoracocentesis, pleurectomy, and chest tube drainage.

Question 57.
A 50-year-old man with chronic pulmonary tuberculosis is brought to the emergency room with worsening cough and dyspnea. A chest X-ray shows features that indicate pleural effusion. Which of the following is not a treatment for pleural effusion?
(a) Labetalol.
(b) Pleurectomy.
(c) Chest tube drainage.
(d) B and C.
Answer:
(a) Labetalol.

Labetalol is not used in the management of pleural effusion. Labetalol is a beta-blocker used in the management of hypertensive urgency and emergency. However, all of the other options are used to manage pleural effusion. Pleural effusion is treated by treating underlying etiology, chest tube drainage, or thoracocentesis.

Question 58.
A 58-year-old patient with diabetes, hypertension, and asthma develops a tension pneumothorax. She presents with headache, chest pain, and increased thirst and urination. A chest X-ray shows features that indicate tension pneumothorax. Which of the following features suggest tension pneumothorax when seen on a chest X-ray?
(a) Mediastinal shift to the contralateral side.
(b) Homogenous opacity.
(c) Patchy opacity.
(d) Hilar fullness.
Answer:
(a) Mediastinal shift to the contralateral side.

Tension pneumothorax is the accumulation of air in the pleural space with cardiopulmonary compromise (mediastinal shift to the contralateral side). It is an emergency and treated with an immediate needle thoracocentesis. Generally, pneumothorax includes primary pneumothorax, which occurs in young men with a tall and thin habitus. There is typically no underlying lung disease. Secondary spontaneous pneumothorax occurs in patients with underlying pulmonary pathology. Traumatic pneumothorax is due to blunt or penetrating trauma to the chest wall. Iatrogenic pneumothorax occurs due to surgical interventions like thoracocentesis, transthoracic needle aspiration, and mechanical ventilation.

Question 59.
Which of the following is true about tension pneumothorax?
(a) It may be caused by trauma to the chest wall.
(b) It always occurs in patients with underlying pulmonary disease.
(c) It is characterized by homogenous opacity on a chest X-ray.
(d) It is treated by surgery.
Answer:
(a) It may be caused by trauma to the chest wall.

Tension pneumothorax may be caused by trauma to the chest wall, especially penetrating chest trauma. Chest trauma can cause a breach in the pleural space, which can lead to air buildup and pressure on the mediastinum. The pressure progressively worsens and causes cardiopulmonary compromise. This presents with tachypnea and, in severe cases, hypotension and shock.

Question 60.
A 55-year-old man with diabetes, peptic ulcer disease, and chronic obstructive pulmonary disease is brought to the emergency room after his breathing difficulty worsens. The nurse examined and identified features that suggest spontaneous pneumothorax. Which of the following issues most likely predisposed the patient to develop this condition?
(a) Diabetes.
(b) Peptic ulcer disease.
(c) Hypertension.
(d) COPD. 
Answer:
(d) COPD.

Primary spontaneous pneumothorax occurs commonly in young men with a tall and thin habitus, typically without underlying lung disease. Secondary spontaneous pneumothorax occurs in people with underlying pulmonary diseases like COPD. Traumatic pneumothorax occurs due to blunt or penetrating trauma to the chest wall. Iatrogenic pneumothorax occurs due to surgical interventions like thoracocentesis, transthoracic needle aspiration, and mechanical ventilation. 

Question 61.
A 10-year-old boy is newly diagnosed with asthma. The parents want information about the disease, its treatments, complications, and how to avoid exacerbations. Which of the following drugs is not used in the management of asthma?
(a) Salbutamol.
(b) Inhaled corticosteroids.
(c) Ipratropium.
(d) Labetalol.
Answer:
(d) Labetalol.

All the other medications are used in bronchial asthma. Labetalol is an antihypertensive. Bronchial asthma is a type one hypersensitivity reaction of the airways, which presents with airway hyperresponsiveness, wheezing, cough, and dyspnea. Patients with mild to moderate cases experience chest tightness, breathlessness, wheezing, and cough. Symptoms often worsen during sleep. On presentation, signs include wheezing, tachypnea, pulsus paradoxus, tachycardia, and breathlessness, evidenced by the use of the accessory muscles of respiration. Patients with severe exacerbations can present with altered consciousness, cyanosis, and a silent chest. In chronic cases, patients have barrel-shaped chests and hyperinflated lungs.

Question 62.
A 28-year-old patient comes into the emergency room in respiratory distress. He reports a history of bronchial asthma. Which of the following medications would be a first-line treatment to relieve his acute symptoms?
(a) Furosemide.
(b) Salbutamol.
(c) Morphine.
(d) Meto]prolol.
Answer:
(b) Salbutamol.

Salbutamol (also known as albuterol) is a short-acting beta-2-adrenergic receptor agonist. It works by relaxing the smooth muscles of the bronchi, which leads to bronchodilation and relief of bronchospasm in conditions like bronchial asthma. This makes it a first-line treatment for acute exacerbations of asthma.

Question 63.
A 17-year-old boy is brought to the emergency department after a severe asthma exacerbation. He is stabilized and nebulized with a medication that is said to be a short-acting beta-2-receptor agonist. Which of the following is not used to evaluate and diagnose bronchial asthma?
(a) Peak flow meter.
(b) Spirometer.
(c) Nebulizer.
(d) Chest X-ray.
Answer:
(c) Nebulizer.

A nebulizer is used to treat, not evaluate, bronchial asthma. A nebulizer is a medical device that converts drugs into aerosol/mists, which makes them easier to inhale, ft is used to nebulize salbutamol, ipratropium, and other drugs to treat acute severe asthma.

Question 64.
Which of the following etiologies would be considered in a patient with acute non-cardiogenic pulmonary edema?
(a) Heart failure.
(b) Pericarditis.
(c) Dilated cardiomyopathy.
(d) Acute respiratory distress syndrome.
Answer:
(d) Acute respiratory distress syndrome.

Acute non-cardiogenic pulmonary edema is the accumulation of fluids in the interstitium of the lungs. It leads to cough, wheezing, and respiratory distress. Acute respiratory distress syndrome includes the following: acute onset of symptoms, respiratory distress, reduced saturation, and chest X-ray showing diffuse interstitial infiltrates. It is managed with mechanical ventilation and treatment of underlying etiology.

Question 65.
Which of the following investigations is appropriate for the initial workup of a patient with acute pulmonary edema?
(a) Echocardiogram.
(b) Chest X-ray.
(c) EKG.
(d) Cardiac enzymes. 
Answer:
(b) Chest X-ray.

The appropriate investigation for the initial workup of acute pulmonary edema is a chest X-ray. Chest X-rays provided information relevant to the diagnosis of pulmonary edema. There is a presence of ground-glass opacities on the chest X-ray for non-cardiogenic pulmonary edema. For cardiogenic pulmonary edema, there are other features suggestive of left ventricular failure. 

Question 66.
Which of the following nursing actions can worsen a patient’s clinical condition with acute pulmonary edema?
(a) Placing the patient in Fowler’s position.
(b) Intranasal oxygen.
(c) Diuretics.
(d) Intravenous fluids.
Answer:
(d) Intravenous fluids.
 
Patients with pulmonary edema should use intravenous fluids with caution. Pulmonary edema is the accumulation of fluids in the interstitium of the lung. It leads to symptoms of dyspnea and cough. Intravenous fluids can worsen pulmonary edema by increasing seepage of fluids into the pulmonary interstitium. Pulmonary edema is managed with oxygen therapy and treatment of the underlying etiology.

Question 67.
A 62-year-old male comes to the ER and mentions increased shortness of breath over the past few weeks. He has a 40-year smoking history. Auscultation reveals decreased breath sounds with prolonged expiration. Which of the following findings is most consistent with a diagnosis of emphysema?
(a) Productive cough with greenish sputum.
(b) Wheezing during both inspiration and expiration.
(c) Fine crackles are heard at the lung bases.
(d) Barrel-shaped chest on physical examination.
Answer:
(d) Barrel-shaped chest on physical examination.

A barrel-shaped chest is a classic physical finding in patients with emphysema. It occurs due to the hyperinflation of the lungs and the flattening of the diaphragm.

Question 68.
Which of the following conditions is often related to deep venous thrombosis?
(a) Heart failure.
(b) Immunosuppression.
(c) Acute pulmonary embolism.
(d) Asthma.
Answer:
(c) Acute pulmonary embolism.

Deep venous thrombosis (DVT) is a risk factor for acute pulmonary embolism (PE). Acute pulmonary embolism is a life-threatening condition caused by obstruction of the pulmonary artery or one of its branches by an embolus. It leads to reduced blood flow to the lungs. This ’causes tachypnea and obstructive shock.

Question 69.
When treating chronic obstructive pulmonary disease (COPD), which of the following medications works by blocking muscarinic receptors and preventing bronchoconstriction?
(a) Fluticasone.
(b) Ipratropium.
(c) Prednisone.
(d) Ceftriaxone.
Answer:
(b) Ipratropium.

Ipratropium is a muscarinic antagonist (also termed anticholinergic). It works by blocking muscarinic receptors in the bronchi, which results in bronchodilation. This mechanism of action is beneficial in conditions like COPD, where chronic bronchoconstriction is a problem. Other medications are used for COPD, but ipratropium specifically has this anticholinergic action.

Question 70.
A 65-year-old patient in the emergency room presents with sudden shortness of breath, pink frothy sputum, and audible crackles on lung auscultation. Which of the following conditions is the most likely diagnosis?
(a) Pneumothorax.
(b) Chronic obstructive pulmonary disease (COPD).
(c) Pulmonary embolism.
(d) Pulmonary edema. 
Answer:
(d) Pulmonary edema.

Pulmonary edema refers to fluid accumulation in the lungs’ alveoli (air sacs). The symptoms commonly include sudden shortness of breath, pink frothy sputum, and crackles on lung auscultation. These symptoms distinguish it from other respiratory conditions.

Question 71.
A 30-year-old driver is brought to the emergency room after he was involved in a traffic accident. He sustained head, chest, abdomen, and limb injuries. A chest X-ray showed the fracture of some ribs. Fracture of three or more adjacent ribs is called which of the following?
(a) Flail ribs.
(b) Flail chest.
(c) Flail sternum.
(d) Unstable ribs.
Answer:
(b) Flail chest.

The fracture of three or more adjacent ribs is called flail chest. This is a serious condition where a segment of the chest wall becomes detached due to multiple rib fractures. This condition can cause paradoxical chest movements during breathing and impair lung function. This can lead to respiratory compromise. Adequate pain management, respiratory support, and treatment of associated injuries are crucial in the management of flail chest.

Question 72.
Which of the following is not a management option for flail chest?
(a) Analgesics.
(b) Oxygen therapy.
(c) Diuretics.
(d) Surgery.
Answer:
(c) Diuretics.

In the initial management of flail chest, opioid analgesics are used to alleviate pain and improve respiratory function. Adequate pain control is essential to prevent respiratory splinting and to promote deep breathing and coughing, which are vital to prevent complications like pneumonia and atelectasis. Oxygen therapy is also important. Surgery is done for severe cases not amenable to medical therapy.

Question 73.
A 46-year-old woman is brought to the emergency room with several injuries. Blood pressure at presentation is 90/50 mm Hg. The patient is in pain and respiratory distress. She is evaluated to rule out life-threatening conditions. Accumulation of blood within the pleural space due to trauma is called which of the following?
(a) Hemothorax.
(b) Hemopericardium.
(c) Empyema.
(d) Pulmonary edema.
Answer:
(a) Hemothorax.

Accumulation of blood within the pleural space due to trauma is called hemothorax. Chest tube drainage evacuates the blood and restores normal lung function. This intervention helps prevent complications such as pleural adhesions and infection.

Question 74.
A nurse is educating a patient newly diagnosed with emphysema about lifestyle modifications. Which of the following statements by the patient indicates a need for further education?
(a) “I’ll make sure to get my flu vaccine every year.”
(b) “I’ve decided to join a smoking cessation program.”
(c) “It’s okay for me to use over-the-counter cough suppressants regularly.”
(d) “I will start attending pulmonary rehabilitation sessions.”
Answer:
(c) “It’s okay for me to use over-the-counter cough suppressants regularly.”

Patients with Emphysema should be cautious about using over-the-counter cough suppressants. These medications can suppress the urge to cough, which is essential for clearing mucus from the airways. Accumulation of mucus can increase the risk of respiratory infections and worsen respiratory function.

Question 75.
A 75-year-old male patient with a history of congestive heart failure is admitted to the ER. The nurse observes a pattern of respiration characterized by increasing depth and frequency followed by a gradual decrease in depth and frequency, which results in a period of apnea. Which of the following best describes this respiratory pattern?
(a) Kussmaul respiration.
(b) Biot’s respiration.
(c) Ataxic respiration.
(d) Cheyne-Stokes respiration. 
Answer:
(d) Cheyne-Stokes respiration.

Cheyne-Stokes respiration is characterized by periodic breathing with a cyclical pattern of increasing and decreasing respiratory rate and depth, followed by a period of apnea. It is often seen in patients with heart failure, stroke, or traumatic brain injuries. The other options represent other abnormal respiratory patterns but do not match the described pattern.

Question 76.
A 50-year-old man is brought to the emergency room following a traffic accident. He was the driver and sustained a blunt chest injury. He is in severe pain and breathless. Chest X-ray shows ipsilateral fracture of three adjacent ribs at more than one point. Which of the following is the most likely diagnosis?
(a) Sternum fracture.
(b) Flail thorax.
(c) Flail chest.
(d) Hemothorax.
Answer:
(c) Flail chest.    

The most likely diagnosis for the 50-year-old man involved in a traffic accident with an ipsilateral fracture of three or more adjacent ribs at multiple points is flail chest. If flail chest is not treated promptly, it can result in respiratory compromise. Management of this patient would typically involve pain relief, respiratory support, and chest wall stabilization.

Question 77.
Opioid analgesics are indicated in the initial management of which of the following conditions?
(a) Pleural effusion.
(b) Spontaneous pneumothorax.
(c) Empyema.
(d) Flail chest.
Answer:
(d) Flail chest.

In the initial management of flail chest, opioid analgesics are essential. These drugs provide pain relief and also improve the respiration of the patient. Pain relief also helps to prevent respiratory splinting. It promotes deep breathing and coughing, which are critical to the recovery of the patient and the prevention of severe complications such as pneumonia and atelectasis. 

Question 78.
A 56-year-old man is brought to the emergency room after suffering chest trauma from a traffic accident. Which of the following is not used in the initial management of a patient with a pulmonary contusion?
(a) Lidocaine.
(b) Supplemental oxygen.
(c) Analgesics.
(d) All of the above.
Answer:
(a) Lidocaine.

Lidocaine is not typically used for the management of pulmonary contusion. Instead, supplemental oxygen should be provided to improve the oxygenation of damaged lung tissue. Analgesics also help provide pain relief and enhance the patient’s respiratory efforts.

Question 79.
A 75-year-old woman was diagnosed with chronic pulmonary hypertension. Which of the following complications might she experience due to this condition?
(a) Right ventricular failure.
(b) Left ventricular failure.
(c) Pleural effusion.
(d) Hydrothorax.
Answer:
(a) Right ventricular failure.

Chronic pulmonary hypertension is a condition that increases blood pressure in the pulmonary arteries. It can lead to complications such as right ventricular failure. This can cause the right ventricle to weaken due to the increased demand and pressure in the pulmonary circulation. This condition requires prompt intervention to ameliorate symptoms and improve heart function.

Question 80.
A nursing student is confused about cor pulmonale. Which of the following is the simplest way to describe this condition?
(a) Left ventricular failure from pulmonary disease.
(b) Right ventricular failure from pulmonary disease.
(c) Left ventricular failure from systemic disease.
(d) Right ventricular failure from systemic disease.    
Answer:
(b) Right ventricular failure from pulmonary disease.

Cor pulmonale is right ventricular failure that results from pulmonary disease, specifically from chronic pulmonary hypertension. It is important to know that this condition is not the same as left ventricular failure. Left ventricular failure is a condition that arises from systemic disease.

Question 81.
A 40-year-old woman is brought to the urgent care clinic after she experienced some gait instability, hypertonia, and paresthesia. She had an episode of optic neuritis a month earlier. She was diagnosed with multiple sclerosis. Multiple sclerosis primarily affects which parts of the brain and spinal cord?
(a) Gray matter.
(b) White matter.
(c) Ventricles.
(d) Peripheral nerves.
Answer:
(b) White matter.

Multiple sclerosis (MS) is caused by degeneration in the brain’s white matter. The white matter comprises nerve fibers covered by the myelin sheath. Multiple sclerosis is an autoimmune disorder characterized by damage to the myelin sheath around nerves in the CNS. The autoimmune reaction disrupts nerve signaling. This causes a wide range of neurological symptoms.

Question 82.
Multiple sclerosis causes the destruction of neurons by autoantibodies in which of the following?
(a) Red blood cells.
(b) White blood cells.
(c) Neuron cell bodies.
(d) Glial cells.
Answer:
(d) Glial cells.

Multiple sclerosis (MS) is an autoimmune disorder where the body’s immune system attacks the myelin sheath, which insulates nerve fibers in the white matter of the brain and spinal cord. The white matter consists of nerve fibers enveloped by the myelin sheath, produced and maintained by glial cells known as oligodendrocytes. Damage to this protective sheath in MS disrupts nerve signaling, which leads to a variety of neurological symptoms. 

Question 83.
A senior emergency nurse teaches nursing students the differences between upper and lower motor neuron lesions. Which of the following disorders do not present with lower motor neuron symptoms?
(a) Poliomyelitis.
(b) Acute inflammatory demyelinating polyneuropathy.
(c) Vitamin B12 deficiency.
(d) Multiple sclerosis.
Answer:
(d) Multiple sclerosis.

Multiple sclerosis primarily presents with upper motor neuron symptoms. The upper motor neuron symptoms include hypertonia and hyperreflexia. Multiple sclerosis (MS) is caused by degeneration of the brain’s white matter. The white matter consists of nerve fibers covered by the myelin sheath. Multiple sclerosis is an autoimmune disorder characterized by autoantibodies in the neurons of the CNS. The autoimmune reaction disrupts nerve signaling. This causes a wide range of neurological symptoms.

Question 84.
A 65-year-old woman with poorly controlled diabetes and hypertension is brought to the emergency room after a stroke. After examination, she is found to have some upper motor neuron deficits in some muscle groups. Which of the following is a characteristic of upper motor neuron lesions?
(a) Hypotonia.
(b) Hyporeflexia.
(c) Hypertonia.
(d) Fasciculations.
Answer:
(c) Hypertonia.

Hypertonia, hyperreflexia, and positive Babisnki reflex are all features of upper motor neuron lesions. In contrast, lower motor neuron lesions cause hypotonia, hyporeflexia, and flaccid paralysis.

Question 85.
Which of the following disorders does not belong to the group?
(a) Vitiligo.
(b) Multiple sclerosis.
(c) Type One Diabetes Mellitus.
(d) Atopic dermatitis. 
Answer:
(d) Atopic dermatitis.

Atopic dermatitis (eczema) is primarily an allergic condition associated with type one hypersensitivity reactions. Type One Diabetes Mellitus is associated with Type Two hypersensitivity reactions. Multiple sclerosis is an autoimmune condition that affects the CNS. Vitiligo is also an autoimmune disorder that affects skin pigmentation. Thus, in terms of hypersensitivity classifications, atopic dermatitis stands out from the group.

Question 86.
A researcher is studying the effect of medications in lowering the auto-antibodies seen in myasthenia gravis. Which of the following receptors are affected by myasthenia gravis?
(a) Presynaptic.
(b) Postsynaptic.
(c) Presynaptic and postsynaptic.
(d) Synaptic cleft.
Answer:
(b) Postsynaptic.

Myasthenia gravis is an autoimmune disease characterized by autoantibodies to the postsynaptic acetylcholine receptors. This leads to muscle weakness, which starts near the eyes. Eyelid weakness is one of the earliest manifestations. The weakness worsens as the day progresses. The condition then progresses to involve the limbs. Involvement of the lungs leads to life-threatening complications known as myasthenia crisis.

Question 87.
Which of the following symptoms is associated with myasthenia gravis?
(a) Muscle rigidity.
(b) Hyperreflexia.
(c) Hypertonia.
(d) Hypotonia.
Answer:
(d) Hypotonia.

Fluctuating muscle weakness, not hypotonia, is one of the hallmarks of myasthenia gravis. The first muscle groups affected are the eyelid muscles. Involvement of the respiratory muscles leads to hypoventilation, CO2 retention, and respiratory failure.

Question 88.
A 68-year-old patient is admitted to the emergency room for myasthenia crisis. Which of the following; vitals is of the highest prognostic importance?
(a) Pulse rate.
(b) Respiratory rate.
(c) Blood pressure.
(d) Temperature.
Answer:
(b) Respiratory rate.

In a patient with myasthenia crisis, monitoring the respiratory rate is of the highest prognostic importance. Respiratory failure can occur due to weakness of the respiratory muscles. Early detection of respiratory distress is crucial for prompt intervention and support.

Question 89.
The treatment for myasthenia gravis includes which of the following?
(a) Plasma exchange.
(b) Blood transfusion.
(c) Intravenous fluids.
(d) Neurosurgery.
Answer:
(a) Plasma exchange.

Plasma exchange, or plasmapheresis, is an effective treatment option to manage severe myasthenia gravis and myasthenia crisis. Plasma exchange removes the autoantibodies against the postsynaptic acetylcholine receptors. This improves the symptoms, returns muscle strength, and healthy pulmonary function.

Question 90.
Which of the following drugs is used to manage myasthenia gravis?
(a) Labetalol.
(b) Thiazides.
(c) Pyridostigmine.
(d) Atropine.
Answer:
(c) Pyridostigmine.

Pyridostigmine is a cholinesterase inhibitor commonly used in the management of myasthenia gravis. By blocking the cholinesterase, an enzyme that breaks down acetylcholine, pyridostigmine causes the accumulation of acetylcholine in the synaptic junction. This enhances nerve transmission and temporarily improves muscle strength.

Question 91.
While examining a 48-year-old patient with a neurologic disease, a nurse notices the patient has ascending flaccid paralysis of some groups of muscles. Which of the following conditions presents with flaccid paralysis?
(a) Transverse myelitis.
(b) Multiple sclerosis.
(c) Guillain-Barre syndrome.
(d) Measles. 
Answer:
(c) Guillain-Barre syndrome.

This patient with ascending flaccid paralysis and a history of paresthesia (neurological disease) most likely has Guillain-Barre syndrome. Guillain-Barre syndrome is characterized by rapidly ascending paralysis. Flaccid paralysis often starts from the lower extremities and progresses upwards.

Question 92.
Which of the following statements is correct about myasthenia gravis?
(a) It presents with ascending and descending spastic paralysis.
(b) It presents with ascending spastic paralysis.
(c) It presents with ascending flaccid paralysis.
(d) It presents with hypotonia.
Answer:
(d) It presents with hypotonia.

Myasthenia gravis (MG) is an autoimmune neuromuscular junction disorder. It is caused by autoimmune involvement of the postsynaptic acetylcholine receptor, impairing nerve signal conduction. The most common symptoms are ptosis, double vision, and muscle weakness that worsens as the day progresses. Treatment is by pyridostigmine or plasmapheresis in severe cases. Thymectomy has also been shown to help with the ~ condition because myasthenia gravis is associated with thymomas.

Question 93.
A 35-year-old man is brought to the ER after developing ascending muscle paralysis and difficulty breathing. The patient was treated for a gastrointestinal tract infection caused by Campylobacter jejuni the previous month. Which of the following is a correct management option for Guillain-Barre syndrome?
(a) Plasmapheresis.
(b) Gabapentin.
(c) Pregabalin.
(d) Pyridostigmine.
Answer:
(a) Plasmapheresis.

Plasmapheresis is a treatment option for Guillain-Barre syndrome. It involves removing the patient’s plasma, which may contain harmful antibodies attacking the peripheral nerves, and replacing it with donor plasma or albumin to alleviate the autoimmune response.

Question 94.
Which of the following infections is associated with the onset of Guillain-Barre syndrome?
(a) Streptococcus.
(b) Klebsiella.
(c) Campylobacter.
(d) Corynebacterium.
Answer:
(c) Campylobacter.

Campylobacter jejuni infection, in addition to viral upper respiratory infections, is a leading infection associated with Guillain-Barre syndrome. Guillain-Barre syndrome is a lower motor neuron neurological disorder characterized by immune cross-reaction with nerve tissues, causing demyelination and ascending muscle paralysis. 

Question 95.
A 40-year-old woman is brought to the emergency department after noticing paresthesia and weakness in her lower legs bilaterally, which is worsening and ascending. The patient recently had Campylobacter diarrhea, which was treated. Which of the following is the patient’s most likely diagnosis?
(a) Poliomyelitis.
(b) Rabies.
(c) Myasthenia gravis.
(d) Guillain-Barre syndrome.
Answer:
(d) Guillain-Barre syndrome.

This patient with a history of Campylobacter infection followed by ascending flaccid paralysis likely has Guillain-Barre syndrome. Guillain-Barre syndrome, also known as acute inflammatory demyelinating polyneuropathy, is a lower motor neuron neurological disorder characterized by immune cross-reaction with nerve tissues, causing demyelination and ascending muscle paralysis.

Question 96.
A 36-year-old woman is brought to the emergency unit with worsening respiratory muscle fatigue. How can the emergency nurse prevent respiratory failure in a patient being managed for Guillain-Barre syndrome?
(a) Intranasal oxygen via face mask.
(b) Intranasal oxygen via nasal prongs.
(c) Mechanical ventilation.
(d) Continuous positive airway ventilation.
 Answer:
(c) Mechanical ventilation.

Mechanical ventilation is useful in preventing respiratory failure in patients with Guillain-Barre syndrome. This is because Guillain-Barre syndrome affects the respiratory muscles in severe cases, which leads to hypoventilation and CO2 retention. Retention of CO2 leads to neurologic symptoms, which worsen the patient’s condition. The endpoint is respiratory failure, a leading cause of death in this type of patient.
 
Question 97.
Which of the following conditions causes a primary headache?
(a) Idiopathic intracranial hypertension.
(b) Acute angle-closure glaucoma.
(c) Encephalitis.
(d) Migraine.
Answer:
(d) Migraine.

Migraine is a primary headache characterized by unilateral throbbing lasting from hours to a day, with associated photophobia and phonophobia, followed by complete resolution. Migraine headaches are also associated with nausea and vomiting and are close differentials of subarachnoid hemorrhage. Patients with migraine headaches are treated with Triptans and NSAIDS for acute flare and topiramate for prophylaxis. 

Question 98.
A 34-year-old woman comes to the urgent care clinic for evaluation of a headache. She is not a known hypertensive, diabetic, or chronic kidney disease patient. She has not lost weight. Which of the following headaches can be treated with 100% oxygen?
(a) Migraine.
(b) Tension-type.
(c) Cluster.
(d) Meningitis.
Answer:
(c) Cluster.

Cluster headache is a type of primary headache that is associated with sensations of tearing and lacrimation. There are also associated autonomic signs such as Horner syndrome. Cluster headaches occur in severe episodes that can be aborted with 100% oxygen.

Question 99.
Which of the following medications is not used to treat migraine headaches?
(a) Triptans!
(b) Ergotamine.
(c) Corticosteroids.
(d) NSAIDS.    
Answer:
(c) Corticosteroids.

All listed options, apart from corticosteroids, are used in the management of migraine. Migraine is a primary headache disorder characterized by abnormal dilatation in cranial microvasculature, leading to throbbing unilateral headache associated with photophobia and phonophobia.

Question 100.
A 30-year-old man is admitted to the emergency unit with complaints of severe headache, vomiting, neck stiffness, and fever for the past three days. Kernig’s and Brudzunski’s signs are both positive. Which of the following is the most likely cause of the headaches?
(a) Tension-type headache.
(b) Migraine headache.
(c) Cluster headache.
(d) Meningitis.
Answer:
(d) Meningitis.

The combination of fever, vomiting, neck stiffness, and positive Kernig’s and Brudzinski’s signs is a likely indication of meningitis. Meningitis is the inflammation of the brain’s covering called the meninges. Meningitis can be secondary to different etiologies, including bacterial, viral, and fungal infections. Diagnosis is by CSF analysis. 

Question 101.    
Idiopathic intracranial hypertension, a cause of headache, is most common in which demographic?
(a) Males.
(b) Neonates.
(c) Females.
(d) Infants. 
Answer:
(c) Females.

Idiopathic intracranial hypertension (IIH), also known as pseudotumor cerebri, is most commonly seen in females, particularly those who are overweight or obese. IIH is characterized by increased intracranial pressure without any structural abnormalities or tumors.

Question 102.
Which headaches are associated with stress and anxiety?
(a) Tension-type.
(b) Migraine.
(c) Cluster.
(d) Idiopathic intracranial hypertension.
Answer:
(a) Tension-type.

Tension-type headaches, in addition to migraine and cluster headaches, are one of the main kinds of headaches. Tension-type headaches are associated with stress and anxiety and are more common in people with emotional, biological, or psychological stressful events.

Question 103.
Which of the following is the normal intracranial pressure?
(a) 1-2 mmHg.
(b) 3-4 mmHg.
(c) 22-30 mmHg.
(d) 7-15 mmHg.
Answer:
(d) 7-15 mmHg.

The normal intracranial pressure is from 7 to 15 mmHg. The intracranial pressure is in equilibrium with the cerebral perfusion pressure and mean arterial pressure. A change in any of the three pressures leads to a change in the others. Raised intracranial pressure is seen in tumors, infections, and stroke.

Question 104.
A patient with raised intracranial pressure can be managed with which of the following drugs?
(a) Propranolol.
(b) Labetalol.
(c) Intravenous fluids.
(d)  Acetazolamide.
Answer:
(d) Acetazolamide.

Acetazolamide is a diuretic used in managing raised intracranial pressure. It is a carbonic anhydrase inhibitor used widely to reduce intracranial pressure. Osmotic diuretics such as mannitol are also commonly used.

Question 105.
All of the following can raise the intracranial pressure, except:
(a) Normal pressure hydrocephalus.
(b) Meningitis.
(c) Encephalitis.
(d) Intracranial space-occupying lesions.
Answer:
(a) Normal pressure hydrocephalus.

Normal pressure hydrocephalus does not cause raised intracranial pressure. It causes ventriculomegaly with normal pressure. It presents as a triad of dementia, urinary incontinence, and gait abnormalities.

Question 106.
A 12-day-old is brought to the neonatal intensive care unit with complaints of irritability and fever. CSF analysis confirms meningitis. Which of the following organisms is a common cause of meningitis in neonates?
(a) Group B streptococcus.
(b) Neisseria meningitides.
(c) Streptococcus pneumonia.
(d) Haemophilus influenzae.
Answer:
(a) Group B streptococcus.

Group B streptococcus is a common cause of meningitis in neonates. Neonates are more susceptible to bacterial infections due to their immature immune systems.

Question 107.
Cheyne-Stokes respiration may be associated with which of the following conditions?
(a) Metabolic acidosis secondary to diabetic ketoacidosis.
(b) Increased intracranial pressure.
(c) Opioid overdose.
(d) Asthma exacerbation.
Answer:
(b) Increased intracranial pressure.

Cheyne-Stokes respiration can be seen in patients with conditions that affect the brain, such as increased intracranial pressure, strokes, and traumatic brain injuries. Although opioid overdose can cause respiratory depression and potentially lead to abnormal respiratory patterns, it does not typically cause Cheyne-Stokes respiration. Metabolic acidosis due to diabetic ketoacidosis typically presents with Kussmaul respirations. Asthma exacerbation leads to a different pattern of respiratory distress. 
 
Question 108.
Which of the following statements correctly describes encephalitis?
(a) Inflammation of the brain parenchyma.
(b) Inflammation of the coverings of the brain.
(c) Inflammation of the spinal cord.
(d) Inflammation of the eyes.
Answer:
(a) Inflammation of the brain parenchyma.

Encephalitis is characterized by inflammation of the brain parenchyma, which refers to the functional tissue of the brain. It is usually caused by viral infections, and symptoms may include headache, fever, altered mental status, and neurological deficits.

Question 109.
A 52-year-old male patient with a history of liver cirrhosis presents to the emergency unit with hematemesis. The physician recommends endoscopic banding. The primary indication for this procedure is:
(a) Gastric ulcers.
(b) Duodenal perforation.
(c) Esophageal varices.
(d) Cholecystitis.
Answer:
(c) Esophageal varices.

Endoscopic banding is also known as variceal band ligation. This procedure is primarily used to treat esophageal varices, which are dilated veins in the lower part of the esophagus. These varices can rupture and bleed, especially in patients with liver cirrhosis. Banding helps to control the bleeding since the bands are placed around the varices to stop the blood flow.

Question 110.    
A nurse is preparing a patient for an endoscopic banding procedure. Which of the following is an essential pre-procedure nursing intervention?
(a) Administering a bowel cleansing enema.
(b) Ensuring the patient is on a clear liquid diet for 24 hours prior.
(c) Ensuring the patient has fasted for 6-8 hours.
(d) Starting an intravenous antibiotic prophylaxis.
Answer:
(c) Ensuring the patient has fasted for 6-8 hours.

Before undergoing endoscopic procedures, including endoscopic banding, the stomach needs to be empty. This will help provide a clear view and reduce the risk of aspiration during the procedure. Hence, fasting for 6-8 hours is typically required.

Question 111.
Which of the following drugs is used in treating absence seizures?
(a) Carbamazepine.
(b) Gabapentin.
(c) Tiagabine.
(d) Ethosuximide.
Answer:
(d) Ethosuximide.

Ethosuximide is a medication commonly used in treating absence seizures. In addition to sodium valproate, it helps to correct the abnormal brain electrical activity associated with absence seizures.

Question 112.
A 35-year-old woman presents with severe headaches, which she described as the worst she has ever experienced. What is the most likely etiology of this presenting complaint?
(a) Subarachnoid hemorrhage.
(b) Cardioembolic stroke.
(c) Migraine headache.
(d) Cluster headache. 
Answer:
(a) Subarachnoid hemorrhage.

Subarachnoid hemorrhage is bleeding into the subarachnoid space. This can result from trauma or rupture of berry aneurysms, also called saccular aneurysms of the brain. It presents with a severe headache, which patients often describe as the worst headache they have ever experienced.

Question 113.
Reperfusion therapy is indicated in which of the following conditions?
(a) Hemorrhagic stroke.
(b) Ischemic stroke.
(c) Migraine.
(d) Tension-type headache.
Answer:
(b) Ischemic stroke.

Reperfusion therapy is indicated in ischemic stroke, not hemorrhagic stroke. Reperfusion therapy comprises surgical thrombectomy or the use of fibrinolytics. A common fibrinolytic agent used in the management of ischemic stroke is alteplase.

Question 114.
Which of the following statements about a stroke is correct?
(a) Strokes can be broadly classified as ischemic or hemorrhagic.
(b) Hemorrhagic strokes occur due to rupture of an intracranial vessel.
(c) Hemorrhagic strokes occur due to occlusion of an intracranial vessel by a thrombus.
(d) All of the above are correct.
Answer:
(a) Strokes can be broadly classified as ischemic or hemorrhagic.

Ischemic stroke refers to the sudden onset of neurologic dysfunction due to a blockade in cerebral blood flow. A hemorrhagic stroke is the result of a rupture in the cerebral vessel. 

Question 115.
Which of the following is true about a transient ischemic attack in a 60-year-old woman?
(a) It causes permanent neurologic dysfunction.
(b) It is due to intracranial bleeding.
(c) It causes neurologic deficits followed by complete resolution within 24 hours.
(d) All of the above are correct.
Answer:
(c) It causes neurologic deficits followed by complete resolution within 24 hours.

A transient ischemic attack, unlike a stroke, causes temporary neurologic deficits, with complete recovery occurring within 24 hours. A transient ischemic attack is often a heralding sign of a stroke.

Question 116.
A woman suddenly lost control of her left upper and lower limbs then regained full control within 24 hours. Which of the following is true about her most likely diagnosis?
(a) No permanent neurologic deficits.
(b) Caused by trauma.
(c) Lasts up to 48 hours.
(d) Treated with immediate surgery.
Answer:
(a) No permanent neurologic deficits.

A transient ischemic attack (TIA) does not result in permanent neurologic deficits. It is often referred to as a “mini-stroke” and is considered a warning sign of an increased risk of a subsequent stroke.

Question 117.
Which of the following is not a risk factor for stroke?
(a) Transient ischemic attack.
(b) Trauma.
(c) Diabetes.
(d) Hypertension.
Answer:
(b) Trauma.

By definition, a stroke is a spontaneous event not caused by trauma. However, trauma can cause intracranial bleeding, such as epidural and subdural hemorrhages.

Question 118.
Which of the following conditions does not usually occur due to a head injury?
(a) Epidural hematoma.
(b) Transient ischemic attack.
(c) Subdural hematoma.
(d) Cerebral contusion. 
Answer:
(b) Transient ischemic attack.

A transient ischemic attack, like a stroke, is a spontaneous occurrence by definition. A transient ischemic attack is not caused by trauma. However, trauma can cause intracranial bleeding, including epidural and subdural hemorrhages.

Question 119.
Which of the following clinical signs is not related to acute appendicitis?
(a) Psoas sign.
(b) Obturator sign.
(c) Rovsing’s sign.
(d) Duke’s sign.
Answer:
(d) Duke’s sign.

Acute appendicitis is associated with all the listed signs except Duke’s sign. Acute appendicitis is an inflammation of the appendix. The diagnosis is primarily clinical, and clinical signs such as psoas, obturator, and Rovsing’s help to confirm the diagnosis.

Question 120.
A patient presents to the ER with a sudden onset of abdominal pain and distention. The physician orders an abdominal X-ray. One of the primary reasons for obtaining an abdominal X-ray in this scenario is to rule out:
(a) Appendicitis.
(b) Gastric ulcer.
(c) Gallstones.
(d) Intestinal obstruction.
Answer:
(d) Intestinal obstruction.

An abdominal X-ray is a primary diagnostic tool used to visualize gas patterns and solid structures within the abdomen. An intestinal obstruction might be suspected in a patient who exhibits sudden abdominal pain and distention. Dilated loops of bowel with air-fluid levels might be seen on the X-ray in the case of an obstruction. The other options are not diagnosed solely with an abdominal X-ray.

Question 121.
Which of the following procedures provides definitive treatment for a patient with acute appendicitis?
(a) Cholecystectomy.
(b) Bilateral tubal ligation.
(c) Appendectomy.
(d) Oophorectomy.
Answer:
(c) Appendectomy.

Appendectomy, the surgical removal of the appendix, is the treatment of choice for acute appendicitis. It is one of the most common surgeries performed by general surgeons. The other options are not used for definitive treatment of appendicitis. 

Question 122.
A 45-year-old man with chronic liver disease is found to have esophageal varices on a routine endoscopy. Which of the following is the most severe complication of esophageal varices?
(a) Perforation.
(b) Bleeding.
(c) Sepsis.
(d) Inflammation.
Answer:
(b) Bleeding.

Esophageal varices are enlarged esophageal veins caused by back pressure from portal hypertension. Esophageal varices commonly occur in people with cirrhosis, and the most severe complication is massive upper gastrointestinal bleeding. Esophageal varices can be treated by endoscopic banding.

Question 123.
A 67-year-old patient presents with recurrent episodes of painless lower gastrointestinal bleeding. She has a history of aortic stenosis and chronic kidney disease. What is the most appropriate diagnostic study to identify angiodysplasias as the potential cause?
(a) Abdominal ultrasound.
(b) Barium enema.
(c) Colonoscopy.
(d) Abdominal MRI.
Answer:
(c) Colonoscopy.

Angiodysplasias are often diagnosed using colonoscopy, which provides direct visualization of the gastrointestinal mucosa. Colonoscopy allows for the identification of characteristic vascular ectasias seen in angiodysplasias and can be both diagnostic and therapeutic, as it provides an opportunity for endoscopic coagulation of these lesions.

Question 124.
A 50-year-old patient comes into the emergency department complaining of passing red blood from the rectum for the past two days. He denies abdominal pain or vomiting. What term is commonly used to describe the passage of fresh blood from the rectum?
(a) Melena.
(b) Hematemesis.
(c) Hematochezia.
(d) Epistaxis.
Answer:
(c) Hematochezia.

Hematochezia refers to the passage of fresh, red blood from the rectum and typically suggests a lower gastrointestinal source of bleeding. It should not be confused with melena, which features tarry, black stools indicative of upper gastrointestinal bleeding. Hematemesis indicates vomiting of blood, and epistaxis is a nosebleed. In patients presenting with hematochezia, the most common causes include diverticular disease, angiodysplasia, neoplasms, and inflammatory bowel disease.

Question 125.
Which of the following causes of upper gastrointestinal bleeding occurs in people with liver cirrhosis?
(a) Esophagitis.
(b) Esophageal web.
(c) Esophageal varices.
(d) Peptic ulcer disease.
Answer:
(c) Esophageal varices.

Esophageal varices are a common cause of upper gastrointestinal bleeding in cirrhotic patients. They commonly occur in people with cirrhosis. The most severe complication is massive upper gastrointestinal bleeding. Esophageal varices can be treated by endoscopic banding.

Question 126.
Helicobacter pylori increases the risk of developing which of the following conditions?
(a) Peptic ulcer disease.
(b) Esophageal varices.
(c) Ileitis.
(d) Spontaneous bacterial peritonitis.
Answer:
(a) Peptic ulcer disease.

Helicobacter pylori is associated with the development of gastric and duodenal ulcers. Over 90% of duodenal ulcers are associated with H. pylori infection, and slightly fewer than 90% are associated with gastric ulcers. Treatment of H. pylori infection facilitates ulcer healing.

Question 127.
Which of the following is a definitive treatment for acute cholecystitis?
(a) Cholangitis.
(b) Cholangectomy.
(c) Cholecystectomy.
(d) Appendicectomy.
Answer:
(c) Cholecystectomy.

The definitive treatment for acute cholecystitis is cholecystectomy, which involves surgically removing the gallbladder. Acute cholecystitis is inflammation of the gallbladder, usually caused by gallstones.

Question 128.
Which of the following conditions most commonly results in liver cirrhosis?
(a) Hepatitis A.
(b) Hepatitis E.
(c) Hepatitis C.
(d) H. pylori.
Answer:
(c) Hepatitis C.

Chronic hepatitis C is a viral infection that can lead to chronic liver inflammation and damage over time, eventually progressing to cirrhosis in some individuals. 

Question 129.
Which of the following signs is seen in decompensated liver cirrhosis?
(a) Chest tenderness.
(b) Rovsing’s sign.
(c) Asterixis.
(d) Shawl sign. 
Answer:
(c) Asterixis.

Asterixis, or “flapping tremor,” is a characteristic sign of decompensated liver cirrhosis. It is a motor abnormality where the hands have a jerking, flapping movement when the wrists are extended, indicating hepatic encephalopathy.

Question 130.
Which of the following conditions does not commonly cause chronic liver disease?
(a) Alcoholic liver disease.
(b) Chronic Hepatitis B.
(c) Chronic Hepatitis C.
(d) Hepatitis E.
Answer:
(d) Hepatitis E.

Hepatitis E does not commonly cause chronic liver disease. It is typically an acute and self-limiting infection. Chronic hepatitis E is rare, especially in immunocompetent individuals.

Question 131.
Which of the following is true about Hepatitis A?
(a) It commonly runs a chronic course.
(b) It is most commonly acquired by blood transfusion.
(c) It is acquired by fecal-oral transmission.
(d) Treatment is a liver transplant.
Answer:
(c) It is acquired by fecal-oral transmission.

Hepatitis A is mainly transmitted through the fecal-oral route, usually from contaminated food or water. It is an acute infection that does not commonly progress to a chronic course.

Question 132.
Which of the following is true about Hepatitis E?
(a) It commonly runs a chronic course.
(b) It is most commonly acquired by blood transfusion.
(c) It is acquired by fecal-oral transmission.
(d) Treatment is a liver transplant.
Answer:
(c) It is acquired by fecal-oral transmission.

Similar to hepatitis A, hepatitis E is mainly transmitted through the fecal-oral route, often due to contaminated water. It is typically an acute infection but can be severe in pregnant women.

Question 133.
Which of the following is not a hepatotropic virus?
(a) Hepatitis A virus.
(b) Cytomegalovirus.
(c) Hepatitis E virus.
(d) Coxsackievirus.    
Answer:
(d) Coxsackie virus.

Coxsackie virus is not a hepatotropic virus. It is a member of the enterovirus family and is known to cause various infections, such as hand, foot, and mouth disease, but it does not primarily target the liver.

Question 134.
A patient is concerned about infections that are transmitted via blood transfusion. Which of the following acute viral hepatitis infections should they not be concerned about?
(a) Hepatitis A.
(b) Hepatitis B.
(c) Hepatitis C.
(d) Cytomegalovirus.
Answer:
(a) Hepatitis A.

Hepatitis A is not transmitted through blood transfusion. It is primarily spread through contaminated food or water or close contact with an infected individual.

Question 135.
A 3-year-old girl is brought to the emergency department following complaints of irritability and excessive crying, change in bowel habits, and bloody stool. Which of the following correctly describes intussusception?
(a) It is commonly associated with Peyer’s patches hypertrophy.
(b) It is always associated with a pathological lead point.
(c) It is most common in the elderly.
(d) It is always idiopathic. 
Answer:
(a) It is commonly associated with Peyer’s patches hypertrophy.

Intussusception is a medical condition where a portion of the bowel telescopes into an adjacent bowel. Although it can occur in any age group, it is most common in young children and is associated with hypertrophy of the lymphoid tissue, including Peyer’s patches in the ileum.

Question 136.
A 5-year-old girl is being evaluated for intestinal obstruction. Which of the following is a common lead point for intussusception?
(a) Meckel’s diverticulum.
(b) Appendix.
(c) Duodenum.
(d) Ileum.
Answer:
(a) Meckel’s diverticulum.

Meckel’s diverticulum is a congenital anomaly of the midgut. It is a common cause of lower gastrointestinal bleeding. It also serves as a pathological lead point for intussusception. 

Question 137.
Intussusception is described as which of the following?
(a) Inflammation of the stomach.
(b) Telescoping of a portion of the bowel into an adjacent bowel.
(c) Hernia of the bowel through a body cavity.
(d) Inflammation of the small intestine.
Answer:
(b) Telescoping of a portion of the bowel into an adjacent bowel.

Intussusception is a common cause of intestinal obstruction in children. It occurs due to invagination of a portion of the bowel into an adjacent layer. Treatment can be nonsurgical or surgical.

Question 138.
A nurse is reviewing an abdominal X-ray report of a pediatric patient which mentions “coin-like structures” in the stomach. The nurse interprets this finding as most likely indicative of:
(a) Polyps.
(b) Foreign body ingestion.
(c) Diverticulitis.    
(d) Fatty liver.
Answer:
(b) Foreign body ingestion.

Children often ingest foreign objects, and coins are among the most common foreign bodies swallowed. An abdominal X-ray can clearly show these coin-like structures in the stomach or intestines, depending on how far they have traveled through the digestive tract.

Question 139.
Hypokalemia causes which type of intestinal obstruction?
(a) Mechanical.
(b) Gallstone ileus.
(c) Bowel edema.
(d) Paralytic ileus.
Answer:
(d) Paralytic ileus.

Paralytic ileus is a form of intestinal obstruction caused by loss of contractile force of the bowel musculature. It can be caused by electrolyte abnormalities such as hypokalemia. It then causes intestinal obstruction. Treatment is by electrolyte correction.

Question 140.
Which of the following is a gastrointestinal complication of hypokalemia?
(a) Perforation.
(b) Bleeding.
(c) Ileus.
(d) Ulceration.
Answer:
(c) Ileus.

Ileus is a disruption of the normal propulsive ability of the gastrointestinal tract and can be caused by various factors, including electrolyte abnormalities. Hypokalemia is commonly complicated by paralytic ileus, leading to abdominal distension, change in bowel habits, and fatigue.

Question 141.
Which of the following is the definitive management of a patient with intestinal obstruction secondary to mechanical obstruction with a nonviable bowel segment?
(a) Bowel lavage.
(b) Bowel resection and anastomosis.
(c) Bowel enema.
(d) Antibiotics. 
Answer:
(b) Bowel resection and anastomosis.

Treatment of intestinal obstruction is by correcting etiology and restoring bowel continuity when needed. Mechanical intestinal obstruction causes a mechanical barrier that has to be removed. If the bowel is nonviable, resection of the necrotic bowel and anastomosis of the remaining bowel ends are done.

Question 142.
A woman comes to the ER complaining that she was diagnosed with pancreatitis at a previous facility. Which of the following is true concerning the pain of acute pancreatitis?
(a) Epigastric pain that radiates to the back.
(b) Right iliac fossa pain.
(c) Colicky epigastric pain.
(d) Colicky right hypochondrial pain.
Answer:
(a) Epigastric pain that radiates to the back.

Acute pancreatitis is characterized by an acute onset of epigastric pain that radiates to the back. The pain is usually worsened by eating due to the release of autodigestive enzymes, further worsening acute pancreatitis. Diagnosis is by abdominal imaging and measurement of serum lipase.

Question 143.
Gallstone is one of the leading causes of which of the following conditions?
(a) Ileus.
(b) Acute appendicitis.
(c) Acute pancreatitis.
(d) Heart failure.
Answer:
(c) Acute pancreatitis.

Gallstones are one of the leading causes of acute pancreatitis. Gallstones cause acute pancreatitis by obstructing the pancreatic ductus, leading to autodigestion of the pancreas by pancreatic enzymes. 

Question 144.
Which of the following best describes the treatment of acute pancreatitis?
(a) Supportive.
(b) Surgical.
(c) Palliative.
(d) All of the above.
Answer:
(a) Supportive.

The treatment of acute pancreatitis is primarily supportive. This includes fasting to rest the pancreas, intravenous fluids to maintain hydration, pain management, and close monitoring for any complications that may arise.

Question 145.
Grey Turner’s sign is a clinical sign associated with which of the following conditions?
(a) Acute appendicitis.
(b) Ovarian torsion.
(c) Acute pancreatitis.
(d) Acute pericarditis.
Answer:
(c) Acute pancreatitis.

Grey Turner’s sign is a clinical sign associated with acute pancreatitis. It is caused by bruising in the flanks, which signifies retroperitoneal hemorrhage and bleeding from the pancreas.

Question 146.
Which of the following conditions might a physician be monitoring if they order repeated partial thromboplastin time (PTT) tests?
(a) Vitamin K deficiency.
(b) Intrinsic pathway clotting factor deficiencies.
(c) Liver disease affecting clotting factor synthesis.
(d) Thrombocytopenia.
Answer:
(b) Intrinsic pathway clotting factor deficiencies.

Partial thromboplastin time (PTT) tests assess the intrinsic and common pathways of coagulation. If PTT is prolonged, it may indicate a deficiency in one or more of the clotting factors in the intrinsic pathway. Liver disease, vitamin K deficiency, and thrombocytopenia are not directly monitored using the PTT test alone.

Question 147.
Which of the following findings points to internal abdominal bleeding in a trauma patient?
(a) Hypotension.
(b) Hypertension.
(c) Weakness.
(d) Rebound tenderness.
Answer:
(a) Hypotension.

Hypotension, or low blood pressure, can be a pointer to internal abdominal bleeding in a trauma patient. Internal bleeding can lead to significant blood loss, causing a drop in blood pressure.

Question 148.
Which of the following organisms is the most common cause of urinary tract infection?
(a) Staphylococcus aureus.
(b) Proteus.
(c) E. coli.
(d) Klebsiella pneumoniae.
Answer:
(c) E. coli.

Escherichia coli (E. coli) is the most common cause of urinary tract infections (UTIs). Jt is a type of bacteria that commonly resides in the gastrointestinal tract and can ascend to the urinary tract, leading to infection.

Question 149.
Neisseria gonorrhea and Chlamydia trachomatis are the most common causes of epididymitis in which patient population?
(a) Elderly.
(b) Neonates.
(c) People of reproductive age.
(d) People of all ages.
Answer:
(c) People of reproductive age.

Neisseria gonorrhea and Chlamydia trachomatis are the most common causes of epididymitis in people of reproductive age, especially sexually active individuals.

Question 150.
Which of the following is the treatment for testicular torsion in a 23-year-old male?
(a) Surgery.
(b) Supportive treatment.
(c) Follow-up.
(d) Observation.
Answer:
(a) Surgery.

Testicular torsion is a urologic emergency that occurs due to the twisting of the spermatic cord. The twisting then cuts off the blood supply to the testicle. Treatment of testicular torsion is emergency surgery. 

Question 151.
A 45-year-old man is brought to the ER with complaints of colicky right flank pain, nausea, and vomiting. The pain waxes and wanes. Which of the following is the most likely diagnosis?
(a) Renal stones.
(b) Acute pancreatitis.
(c) Peptic ulcer disease.
(d) Renal failure.
Answer:
(a) Renal stones.

This patient with colicky flank pain, vomiting, and nausea most likely has renal stones. Renal stones may pass spontaneously with adequate hydration in most cases. However, larger stones may need surgery to be removed. This colicky pain is due to the obstruction caused by the stones.

Question 152.
Which of the following is not a correct management intervention for a male patient presenting to the ER with bladder trauma?
(a) Catheterization.
(b) Cystoscopy.
(c) Surgical repair.
(d) Urethroplasty. 
Answer:
(d) Urethroplasty.

Urethroplasty is a urological procedure used in managing urethral stricture. It is not used for the management of bladder rupture. However, bladder rupture and urethral stricture may coexist.

Question 153.
Which of the following is the most appropriate initial measure for the management of a 60-year-old man presenting with acute urinary retention?
(a) Urgent surgery.
(b) Catheterization.
(c) Intravenous fluids.
(d) Placing on nil per oris.
Answer:
(b) Catheterization.

Acute urinary retention is a common complication of bladder outlet obstruction. It causes pain and suprapubic tenderness. The most important initial management intervention is urethral catheterization after excluding the urethral stricture.

Question 154.
Which of the following is true about pelvic inflammatory disease?
(a) It causes left hypochondrial pain and tenderness.
(b) It is caused mainly by staphylococcus aureus.
(c) It is caused primarily by viruses.
(d) It is an ascending infection of the reproductive tract.
Answer:
(d) It is an ascending infection of the reproductive tract.

Pelvic inflammatory disease is a common gynecological condition among people of reproductive age. It is an ascending infection of the genital tract, which usually progresses from cervicitis to endometritis and oophoritis.

Question 155.
Which of the following conditions is not a common cause of antepartum hemorrhage?
(a) Placental Abruption.
(b) Uterine atony.
(c) Placenta previa.
(d) All of the above.
Answer:
(b) Uterine atony.

The most common cause of postpartum hemorrhage is uterine atony. Uterine atony is a condition in which the muscles of the uterus do not adequately contract after delivery. Thus, the placental vessels are not closed properly and bleeding cannot be prevented. Common causes of antepartum hemorrhage include placental abruption and placenta previa.

Question 156.
A 30-year-old woman presents with elevated blood pressure, 3+ protein and an episode of convulsion. A diagnosis of eclampsia is made. Which of the following is used in the management of eclampsia?
(a) Carbamazepine.
(b) Magnesium sulfate.
(c) Digoxin.
(d) Diazepam.
Answer:
(b) Magnesium sulfate.

Magnesium sulfate is used in seizure prophylaxis and in the management of patients with eclampsia. It can be given through both intramuscular and intravenous routes. It reduces morbidity and mortality for gestational hypertension and associated complications.

Question 157.    
A 35-year-old woman presents with painful, rhythmic uterine contractions before 37 weeks of gestation. This presentation is referred to as which of the following?
(a) Pre-term labor.
(b) Post-term labor.
(c) Spurious labor.
(d) Miscarriage.
Answer:
(a) Pre-term labor.

This patient with painful, rhythmic uterine contractions before 37 completed weeks of gestation most likely has pre-term labor. Pre-term contractions can be caused by infections or maternal and fetal conditions. The goal of management is the prevention of pre-term delivery and its complications. 

Question 158.
Which of the following increases the risk of uterine rupture during labor and delivery?
(a) Primigravidity.
(b) Maternal age less than 30.
(c) Eclampsia.
(d) Previous cesarean section. 
Answer:
(d) Previous cesarean section.

A previous cesarean section is a strong risk factor for uterine rupture. This is true because the uterus is prone to rupture along the lines of the healed scar. Other risk factors for uterine rupture include grand multiparity and uterine anomalies.

Question 159.
A 25-year-old man with a psychiatric history is brought to the ER after he attacked children who were playing at a park. Which of the following is not associated with an increase in the risk of aggression and violent behaviors?
(a) Previous history of violent behaviors.
(b) Mania.
(c) Schizophrenia.
(d) Calcium intake.
Answer:
(d) Calcium intake.

Calcium intake is not a risk factor for violence and aggression in psychiatric patients. A history of psychiatric illness, schizophrenia with commanding auditory hallucinations, and bipolar affective disorder are all risk factors for aggression.

Question 160.
A 12-year-old girl was diagnosed with a panic disorder. Which of the following is true concerning panic disorders?
(a) Affected people are afraid of being separated from their loved ones.
(b) There is an intense episode of fear of an impending adverse occurrence that is associated with palpitations and avoidance of such situations.
(c) Affected patients often avoid social situations and gatherings.
(d) Patients worry excessively in most situations.
Answer:
(b) There is an intense episode of fear of an impending adverse occurrence that is associated with palpitations and avoidance of such situations.

Panic disorders are characterized by an intense episode of fear. Patients often have triggers that cause these panic attacks. Other signs include rapid breathing or hyperventilation.

Question 161.
A woman has an intense fear of enclosed spaces. Which of the following describes an intense fear of that situation?
(a) Depression.
(b) Phobia.
(c) Grief.
(d) Post-traumatic stress disorder.
Answer:
(b) Phobia.

Phobia is a form of anxiety disorder characterized by an intense fear of a particular situation or object. The woman in this question has an intense fear of enclosed spaces. The phobia can also be toward animals, places, or things. The treatment is systematic desensitization and graded exposure.

Question 162.
A 28-year-old woman comes to the urgent care clinic for an evaluation and treatment of her anxiety. Which of the following medications is the first choice for treating generalized anxiety disorders?
(a) Benzodiazepines.
(b) Selective Serotonin Reuptake Inhibitors.
(c) Analgesics.
(d) Anesthetists.
Answer:
(b) Selective Serotonin Reuptake Inhibitors.

Selective serotonin reuptake inhibitors (SSRIs) such as paroxetine, fluoxetine, and „ sertraline are used as first-line medications in the management of generalized anxiety disorders (GADs). These drugs are also the first choice in the management of depression.

Question 163.
Which of the following is true about a patient with a social anxiety disorder?
(a) Affected people are afraid of being separated from their loved ones.
(b) There is an intense episode of fear of an impending adverse occurrence that is associated with palpitations and avoidance of such situations.
(c) Affected patients often avoid social situations and gatherings because of the fear of being judged by others.
(d) Patients worry excessively in most situations. 
Answer:
(c) Affected patients often avoid social situations and gatherings because of the fear of being judged by others.

Social phobia is a specific form of phobia in which an individual avoids social gatherings for the fear that others might judge their manners, attitudes, behavior etc. As a result, the patients avoid social gatherings.

Question 164.
A 35-year-old woman with bipolar affective disorder comes to the ward for evaluation. Which of the following statements correctly describes bipolar affective disorder?
(a) Alternating episodes between mania and depression.
(b) Alternating episodes between mania and anxiety.
(c) Alternating episodes between depression and schizophrenia.
(d) Alternating episodes between depression and anger.
Answer:
(a) Alternating episodes between mania and depression.

Bipolar affective disorder is a mood disorder characterized either by episodes of mania that alternate with depression or by hypomania that alternates with depression. Hypomania is a persistent elated mood that does not meet the criteria of mania (that does not cause a loss of social functioning). 

Question 165.
Which of the following qualifies a patient for the diagnosis of bipolar affective disorder?
(a) Two episodes of mania.
(b) A manic episode followed by a depressive episode.
(c) Depression alternating with hypomania.
(d) All of the above.
Answer:
(d) All of the above.

All of the options are correct. A physician can safely diagnose bipolar affective disorder in any of the following scenarios: if the patient has two episodes of mania, or if the patient has a manic episode followed by a depressive episode, or if the patient has depression alternating with hypomania.

Question 166.
A 35-year-old woman is brought to the ER after an episode of deliberate self-harm. She was said to be experiencing some marital issues because her husband left her. She is tearful, in a low mood and says she feels hopeless. Which of the following is the diagnosis?
(a) Severe mania.
(b) Bipolar affective disorder.
(c) Severe depression.
(d) Severe grief.
Answer:
(c) Severe depression.

The patient who is sad with a history of deliberate self-harm most likely has severe depression. The patient should be admitted and treated as an inpatient to reduce the risk of suicide.

Question 167.
Which of the following is an indication of electroconvulsive therapy?
(a) Anxiety.
(b) Grief.
(c) Phobia.
(d) Severe depression with suicidal ideation.
Answer:
(d) Severe depression with suicidal ideations.

Electroconvulsive therapy uses electrical signals to alter the neurotransmission of information in the brain. It is used to treat severe depression in patients with suicidal ideation. 

Question 168.
Which of the following is an example of a psychotic symptom?
(a) Sadness.
(b) Grief.
(c) Fear.
(d) Delusions. 
Answer:
(d) Delusions.

Delusions and hallucinations are the main examples of psychotic symptoms. Delusion is a result of a thought disorder, which causes a loss of perception of reality. There are different types of delusions, including paranoia, jealousy, and grandiose.

Question 169.
Which of the following drugs is used in the management of the psychotic symptoms of schizophrenia?
(a) Risperidone.
(b) Diazepam.
(c) Sertraline.
(d) Lorazepam.
Answer:
(a) Risperidone.

Risperidone is an antipsychotic medication used in the management of psychotic „ symptoms of schizophrenia. It helps reduce the positive and negative symptoms of the condition. The positive symptoms include delusions and hallucinations. The negative symptoms include apathy and social withdrawal.

Question 170.
Dangerousness in a patient with a psychiatric history is an indication for mental health practitioners. Which of the following actions is most appropriate for a patient who may be dangerous?
(a) Refer the patient to another practitioner.
(b) Get protective gear.
(c) Admit the patient into a mental health facility.
(d) Do a home follow-up for the patient.
Answer:
(c) Admit the patient into a mental health facility.

Upon evaluation and assessment of a patient with a psychiatric history, if findings show they can be dangerous to themselves or others around them, there might be a need to admit the patient into a mental health facility. This would help to provide a safe and enabling environment for more assessment and therapy.

Question 171.
A 5-year-old boy is found lying restless on the floor after taking about 14 tablets of acetaminophen. Which of the following organs is most often damaged by acetaminophen poisoning?
(a) Liver.
(b) Lungs.
(c) Kidney.
(d) Intestine.
Answer:
(a) Liver. 

Acetaminophen poisoning primarily damages the liver. An overdose of acetaminophen can cause liver toxicity and may lead to acute liver failure if not promptly treated.

Question 172.
Which of the following drugs is used in the management of opioid poisoning?
(a) Liquid paraffin.
(b) Diazepam.
(c) Diclofenac.
(d) Naloxone.
Answer:
(d) Naloxone.

Naloxone is used in the management of opioid poisoning. All of the other options listed are not used for managing opioid poisoning.

Question 173.
Opioid overdose can be treated by which of the following medications?
(a) Naloxone.
(b) Flumazenil.
(c) Diazepam.
(d) Calcium gluconate. 
Answer:
(a) Naloxone.

Opioid overdose typically presents with pupillary constriction and respiratory depression. It can be treated with naloxone, which is an opioid receptor antagonist. Naloxone quickly reverses the effects of opioids, including respiratory depression, and can save lives in cases of opioid overdose.

Question 174.
A 35-year-old man comes to the clinic with complaints of double vision and weakness in the limbs. He felt stronger a few years back, but has been progressively feeling weaker recently. Examination reveals a muscle power of 3/5 in both upper limbs. After full evaluation he is found to have autoantibodies against postsynaptic acetylcholine receptors. What is the patient’s diagnosis?
(a) Grave’s disease.
(b) Myasthenia gravis.
(c) Celiac disease.
(d) Lambert-Eaton syndrome.
Answer:
(b) Myasthenia gravis.

This patient with complaints of double vision, limb weakness and a loss of muscle power most likely has myasthenia gravis, which is an autoimmune disorder of the muscles. It is characterized by autoantibodies against postsynaptic acetylcholine receptors, and leads to muscle weakness that worsens as the day progresses. Diplopia and fatigue are also signs. The diagnosis of myasthenia gravis is done by the Tensilon test and it is treated by administering pyridostigmine.

Question 175.
A 40-year-old woman is brought to the ER after having progressive difficulty with breathing for the past few weeks. The patient has no cough or fever. She also reports weakness in the upper and lower limbs that worsen as the day goes by. The patient is diagnosed with myasthenia gravis. Which of the following tests can be used to diagnose this condition?
(a) The Wicjal test.
(b) The Polymerase chain reaction test.
(c) The Tensilon test.
(d) Computed tomography.
Answer:
(c) The Tensilon test.    

The Tensilon or Edrophonium test is used for diagnosing myasthenia gravis. Edrophonium chloride, a cholinesterase inhibitor, is administered and the patient’s muscle power is examined after five minutes. Improvement in the patient’s muscle power confirms the diagnosis of myasthenia gravis. Edrophonium is short-acting, so it is mainly used for diagnosis, not treatment. Pyridostigmine is a long-acting cholinesterase inhibitor, so it is used for treatment. 

Practice Tests:

Book an appointment